You are on page 1of 2182

Pediatrics By Dr.

Anand

MD PEDIATRICS
• Importance of Studying Pediatrics
for exams ?
• How to study ?
• How to study ?
• What to study ?
• How to study ?
• What to study ?
• Where to start ?
•Repetitive
•Repetitive
•To the point
•Repetitive
•To the point
•High scoring subject
• How should I improve my
efficiency for exam ?
• How should i improve my
efficiency for exam ?
• Time table 1 day prior
• Avoid Cramming & start Understanding
•Solve lot of practice papers
•Take short breaks
•Proper meals & rest
Turn your focus towards the process
& away form the outcome
•Be familiar with the structure of the
exam
If you get stuck – MOVE ON !!
Symmetrical Asymmetrical

Time of insult First / early second Late second / 3rd

Etiology TORCH Poor maternal


Genetic nutrition / HTN /
Chromosomal UPI
Effect on cells Fetal cell no. dec. Size > No.

Growth HC / wt / length HC normal


equally affected
Ponderal index > Or equal to 2 <2
Chapter:
NEONATOLOGY
•Important Definition
•Neonate:
•Early neonate:
•Late neonate:
• Neonate: 0-28

• Early neonate: 0-7 days

• Late neonate: 8-28 days


• Infant:
• Toddler:
• Preschool:
• MCC of neonatal
mortality in India:
• MCC of neonatal
mortality in India:

MCC of distress in
• Preterm
• Term
• Post term
• Antenatal steroids ?
• Based on gestational age & birth
weight:
• AGA, SGA, LGA
• Normal birth weight: 2.5-4.0kg

• Average wt – 3 kg

• LBW: <2.5kg

• VLBW: <1.5 kg

• ELBW: <1.0kg

• Big baby: >4 kg


• Term: 37-41 completed weeks

• Preterm: <37w POG

• Post term: >42w POG


• IUGR
Symmetrical Asymmetrical

Time of insult First / early second Late second / 3rd

Etiology TORCH Poor maternal


Genetic nutrition / HTN /
Chromosomal UPI
Effect on cells Fetal cell no. dec. Size > No.

Growth HC / wt / length HC normal


equally affected
Ponderal index > Or equal to 2 <2
•Causes of LGA?

• SBI Bank – Extra account


Causes of LGA ? SBI-E A
• Sotos syndrome / cerebral gigantism

• Beckwith-Wiedemann Syndrome

• IDM

• Erythroblastosis fetalis

• Constitutional
Macroglossia, microcephaly, low set ears
• Pinna abnormalities

• History of polyhydromnios

• Ear lobe creases


• Ear lobe creases / fissure **
• Wilms tumour
•Happy & a healthy mind
• AIIMS NICU Protocol Updates 2022

• A 2.5 kg newborn baby born through LSCS, cried


immediately after birth. On further examination the
mother is HbsAg +ve. Parents are worried about the
further management of the baby. You are the doctor on
duty. What is the best management according to you ?
Q: What is the composition of hepatitis B
immunoglobulin ?
A. 0.5 ml = 100 IU
B. 0.5 ml = 300 IU
C. 0.5 ml = 400 IU
D. 0.5 ml = 600 IU
Q: What is the composition of hepatitis B
immunoglobulin ?
A. 0.5 ml = 100 IU
B. 0.5 ml = 300 IU
C. 0.5 ml = 400 IU
D. 0.5 ml = 600 IU
Q: What is the route of giving hepatitis B
immunoglobulin ?
A. IM
B. SC
C. ID
D. IV
Q: What is the route of giving hepatitis B
immunoglobulin ?
A. IM
B. SC
C. ID
D. IV
Q: within how may hours we must administer hepatitis
B immunoglobulin ?
A. 48-72 hr
B. 72-96 hr
C. Anytime of the day
D. Within 5 days
Q: within how may hours we must administer hepatitis
B immunoglobulin ?
A. 48-72 hr
B. 72-96 hr
C. Anytime of the day
D. Within 5 days
Q: What is the schedule of hepatitis B vaccine ?
•Hepatitis B vaccine

• Given at birth, 6w, 6Mnths or birth,6w, 10w, 14w


Q: hepatitis B vaccine gives __ % of active immunity and with
hepatitis B immunoglobulin give additional ___ % of active immunity
?

A. 70-90% & 10-30%


B. 70-100% & 10-30%
C. 70-90% & 12-30%
D. 70-90% & 10-40%
Q: hepatitis B vaccine gives __ % of active immunity and with
hepatitis B immunoglobulin give additional ___ % of active immunity
?

A. 70-90% & 10-30%


B. 70-100% & 10-30%
C. 70-90% & 12-30%
D. 70-90% & 10-40%
Q: if the mother is hepatitis B +ve, she must not
breast feed the child

A. True

B. False
Q: if the mother is hepatitis B +ve, she must not
breast feed the child

A. True

B. False
•Breast Feeding: Bullet points
•Breast Feeding: Bullet points

•Breast feeding week – important dates


•Breast Feeding: Bullet points

•Breast feeding week

•What is exclusive breast feeding


•Breast Feeding: Bullet points

•Breast feeding week

•What is exclusive breast feeding

•What is weaning
•Breast Feeding: Bullet points

•Breast feeding week

•What is exclusive breast feeding

•What is weaning

•Breast feeding till how many years ?


• Doctor my baby is not feeding well – how to approach ?
• Signs of good attachment

• Chin touching the breast

• Mouth wide open with fullness of cheeks

• Lower lip everted out

• Upper areola exposed out


• NVD

• LSCS
• NVD - ASAP

• LSCS – 4HRS
• Reflexes help in breast feeding
• Prolactin Produces
• Oxytocin Oozes
• Breast milk cannot cause evening colic
•C
•C
•C
•C
•C
• Max breast milk output
occurs at:
• Max breast milk output
occurs at:

• STORE the breast milk at:

• Room temp.
• Max breast milk output
occurs at:

• STORE the breast milk at:

• Room temp.

• Refrigerator

• -20 degree Celsius :


• Max breast milk output
occurs at: 5-6 months, 730
ml per day
• Room temp.-6-8 hrs

• Refrigerator- 24 hrs

• -20 degree Celsius : 3


months
Human Cow’s milk
milk
Protein

Calories

Lactose

Fat

Sodium / K
Human Cow’s milk
milk
Protein 1.1g* 3.5g*/3.0

Calories 65 Kcal 67 Kcal

Lactose 7.4 gm 4.5 gm

Fat 3.8g rich in 3.7 gm


PUFA*
Sodium / K 7 & 14 25, 35
FOREMILK HINDMILK
FOREMILK HINDMILK

BEGINNING OF FEED END OF FEED

WATER VITAMINS FAT


MINERALS PROTEINS

THIRST OF BABY HUNGER OF BABY


• Finish
• Where life plants you
• Where life plants you

• Bloom with grace !!


• Maternal drug contraindicated in
pregnancy ?
• Maternal drug contraindicated in
pregnancy ?

• Aspirin
• Paracetamol
• Insulin
• Maternal drug contraindicated in
pregnancy ?

• Aspirin : reyes syndrome


• Paracetamol
• Insulin
• Benefits of breast feeding
• Benefits of breast feeding

than cow’s milk:

q More *

q Less *

q Richer
• More in Carbohydrates
• Which is the predominant
carbohydrate in breast milk ?
• Which is the predominant
carbohydrate in breast milk ?

• Lactose *
• Which is the predominant
carbohydrate in breast milk ?

• Lactose *
• Glucose + galactose
• Which makes the milk sweety
• Which is the predominant
carbohydrate in breast milk ?

• Lactose *
• Glucose + galactose
• More energy
q More LACTOSE*

q Less PROTEIN*

q Richer in whey protein

(lactalbumin) which are easily

digestible as compared to

cow’s milk which are rich in

casein
• MOTHER MILK IS RICH IN LACTOSE & less in
proteins *

• BUT PRETERM MILK IS LOW IN LACTOSE &


more in proteins**
q Contains good amount of

q Easily digestible due to ?

q Breast milk contains all vitamins

except
q Contains good amount of DHA

(docosa hexaenoic acid) helps in

brain and eye development

q Easily digestible due to presence

of lipase in breast milk


Opposite are deficient in both

• Mother will feed the baby in AC

room
Opposite are deficient in both

• Mother will feed the baby in AC

room

• So mother milk is rich in vitamin A,C


• And also breast milk is

deficient in vitamin D,K &

B12 (pure veg )


• Which injection do we give to a newborn ?
• Which injection do we give to a newborn ?

• Vitamin K

• <1kg – 0.5 mg
• >1 kg – 1 mg
• Which injection do we give to a newborn ?

• Vitamin K

• <1kg – 0.5 mg
• >1 kg – 1 mg

• Syrup vitamin D – 400/800 IU till 1 yr


• IRON – after 4 weeks of life till 1 yr
• Difference b/w human and cow milk
• Some book says 3.0 ( Elizabeth)
• Some says 3.5
• Jipmer 2019

• Fat content in breast milk ?


• Preterm / Term / Cow
• Preterm / Term / Cow

• 2.3 / 1.1 /3.1


• Fatty acid present in breast milk which is important for
growth & CNS development is _________*** ?
• Fatty acid present in breast milk which is important for
growth & CNS development is

DHA ?

Docosa hexaenoic acid


• Human milk contains 2

types of protein 60%

whey and 40% casein

helps in quick and easy

digestion.

• Cow milk: 20:80


Spotter ..
v Colostrum :the first immunization

of the baby

v Golden thick yellow milk during

first 3 days of life

v IgA*, proteins*(as compared to

breast milk), WBCs

v ideal first MEAL for the baby


Remember HSP also it is IgA deposition everywhere
vTransitional milk :

Composition is in

between colostrum and

mature milk
v Transitional milk :

Composition is in between

colostrum and mature milk

v Mature milk: thin, water &

contain less proteins &

more fat + sugar are more


v Transitional milk : Composition is in

between colostrum and mature milk

v Mature milk: thin, water & contain less

proteins

v Preterm milk contains more calories, fat

,protein, Ig which are needed for a

preterm baby
Anti infective factors in breast milk:
To Be in LAP
Anti infective factors in breast milk:

To Be in LAP

v TGF beta: Bio active factors *


Anti infective factors in breast milk:

To Be in LAP

v TGF beta: Bio active factors *

v Bifidus factor & Lactoferrin(iron binding protein)** : Shigella,

salmonella, E.coli
vLactoferrin: G.Lamblia*, P. falciparum,

T gondii, E histolytica – Lactobacillus

which is a gut microflora


v Lactoferrin: G.Lamblia*, P. falciparum, T

gondii, E histolytica – Lactobacillus which

is a gut microflora

v Lysozyme & BSSL (bile salt stimulated

lipase) : E Histolytica*
vAntibodies IgA

vAnti- staphylococcal factor


vPhagocytic macrophages

vPABA: malaria P.VIVAX*


• TGF – transforming growth factor beta
Contraindication of breast feeding
Contraindication of breast feeding

• Galactosemia

• Lactose intolerance

• Mother on Chemo + Radio

• Active untreated TB (relative)


• Mixed feeding is contraindicated in
breast milk

• Bottle + breast
• Breast feeding should be avoided by a
mother on ATT for ___ weeks ?
• Breast feeding should be avoided by a
mother on ATT for 2. weeks ?
Relative
• Maternal HIV: contraindicated in
developed countries, mixed feeding

in developing
• Active untreated TB: breast feeding

is contraindicated until completion of

2 weeks of ATT

• Varicella zoster: infant should not

have direct contact with active

lesions

• Herpes simplex infection**


• Recommended daily energy requirement:

• 0-10 kg:

• 10-20 kg:

• >20 kg:
• Recommended daily energy requirement:

• 0-10 kg: 100Kcal/kg

• 10-20 kg: 50 Kcal/Kg

• >20 kg: 20 Kcal/kg


Ref: Nelson 20/e p 65-77, ghai 8/e p a

Q10: Nipple confusion means: APPG


2014

A. Baby fed with bottle finding it


difficult and confusion to suckles at
breast

B. Baby not able to suck with bottle

C. Baby not able to feed with spoon

D. Baby not able to feed with paladin


Ref: Nelson 20/e p 65-77, ghai 8/e p a

Q10: Nipple confusion means: APPG


2014

A. Baby fed with bottle finding it


difficult and confusion to suckles at
breast

B. Baby not able to suck with bottle

C. Baby not able to feed with spoon

D. Baby not able to feed with paladin


• It’s not how we make mistakes
• It’s not how we make mistakes

But how we correct them that


defines us
Vitals of normal term neonate:
qHR

qRR

qCFT

qAny cyanosis

qAny murmur
Vitals Of Normal Term Neonate:

vHR: 110-160/min (120-160bpm Ghai)

vRR: 40-60 cpm

vCFT: <3 seconds

vAny cyanosis: yes acrocyanosis


Q23: Normal HR in a newborn:

June 2010
Ref: Nelson 20/e p c

A. 80-100

B. 100-120

C. 120-160

D. 160-200
qLength of umbilical cord

qLength of newborn

qHC

qWeight

qUS:LS ratio

qFirst urine &

stool(meconium) by:
vAny murmur: soft ejection systolic murmur

vLength of umbilical cord: 48-50cm

vLength of newborn: 48-50cm

vHC: 33-35cm

vWeight: 2.5-3.8 kg

vUS:LS ratio: 1.7:1


• Normal weight: 2.5-4kg

• Average Indian baby wt.: 3kg* ( ghai)

• Normal length: 48-50cm ( inc. by 50%

at 1st yr=75cm)

• Normal HC: 33-35cm

• Normal CC: 31cm ( -3) HC = CC by 9-

12months
vExpected weight formula

vExpected height formulae

vWeight doubles at:

vWeight TRIPLES at:

vHeight doubles at:


vExpected weight formula:

v<1yr,1-6,6-12yr

vExpected height formulae:

vage * 6 + 77

vWeight doubles at:

v5 months * triple qt 1 yr*, 4times at 2 yr. *

5 TIMES BY 3 YR

vHeight doubles at: 4 yrs


AGE

5 MONTHS Weight doubles of


birth weight

1 YR

10
AGE

5 MONTHS Weight doubles of


birth weight

1 YR TRIPLES

2 4 TIMES

3 5 TIMES****

5 6 TIMES

7 7 TIMES

10 10 TIMES
• Length increases by 50%

In the 1st year of life **

• Height doubles at 4 yr

• triples at 12 yrs.
Q: What is the expected weight of 2 yr old?

RECENT QUESTION 2016

A. 10kg

B. 12 kg

C. 14kg

D. 16 kg
• NORMAL GROWTH Instruments:

• Length:

• Height:

• HC:

• Skin fold thickness:


• NORMAL GROWTH Instruments:

• Length: Infantometer* <2 yr

• Height: Stadiometer* > 2 yr

• HC: non stretchable measuring tape

• Skin fold thickness: HARPENDEN

CALLIPERS*
IDENTIFY THE INSTRUMENT?
To know the caloric reserve
of the body
Measured to the nearest
of 1 cm
Ref: Nelson 20th p 1768,ghai 8/e p11 c

Q2: Which of the following is NOT true in

assessing skin fold thickness:

RECENT MAY 2018

A. Measured at the triceps level

B. Measured to the nearest of 1 cm

C. Tanner’s chart contains the normal value

D. Indirectly indicated caloric reserve of the

body
Height
• At birth:

• 3 month:

• 6 month:

• 9 month:

• 1 yr:

• 2 yr:
• At birth: 50cm *

• 3 month: 60 cm

• 6 month: 65cm

• 9 month: 70cm

• 1 yr: 75cm*

• 2 yr: 90cm
• 4 yr:

• 12 yr:

• Formula:

• 5&4
Height
• 4 yr: 100cm *

• 12 yr: 150 cm

• Formula: age * 6 + 77

• 5 & 4: weight doubles at

4 months, ht. by 5 year


• Upper segment : lower segment ratio

• Why is it important?

• Normal at birth:

• 3 years:

• 7-10 years:
Upper segment : lower segment ratio
Vertex to pubic symphysis

Pubic symphysis to heel

Normal at birth- 1.7:1

3 years: 1.3:1

7-10 years: 1:1


vHEAD CIRCUMFERENCE
0-3 months 2cm/month

3-6 months 1 cm/ month

7-12 months 0.5 cm/ month

1-3 yr 0.2/month
Question
•Life is a journey not a race
Vitals Of Normal Term Neonate:

vFirst urine & stool

by: urine by __ hrs

vStool by __ hours
Vitals Of Normal Term Neonate:

vFirst urine & stool

by: urine by 24 hrs

vStool by 48 hours

vCloherty*
How to differentiate

Pre term & term baby?


Pre term Term Post term
Preterm / term:
1. Scalp hair

2. Breast nodule

3. Genitals

4. Lanugo

5. Palmar creases
How to differentiate
term & post term?

Anyone ?
Peeling of the skin
Q: The sole of a newborn baby shown below is suggestive of:

Term, preterm, neonatal jaundice, LGA baby ?


Main points:

• Tool used to assess gestational age of newborn is

_____________________

• ________ weeks
Main points:

• ________ weeks

A. 10-40 weeks

B. 20-44 weeks

C. 24-44 weeks

D. 40 – 44 weeks
Main points:

• ________ weeks

A. 10-40 weeks

B. 20-44 weeks

C. 24-44 weeks

D. 40 – 44 weeks
Main points:

• Tool used to assess gestational age of newborn is

EXPANDED NEW BALLARD SCORE

• 20-44 weeks
Neuromuscular maturity
Physical maturity
Maturity
Rating
Score Weeks

–10 20
–5 22
0 24
5 26
10 28
15 30
20 32
25 34
30 36
35 38
40 40
45 42
50 44
• JIPMER
• WHICH ONE IS TILL 5 & -2
• Which sign is elicited here?
• Which sign is elicited here?
• Which sign is elicited here?
• To check the TONE **

• Scarf sign

• Elbow crosses the midline


• Can newborn shiver ?

• Does newborn have tears ?

• Are newborns Colour blind ?


Can newborn shiver ?

Ø They don’t shiver

Ø They have non shivering

thermogenesis (NST)

Ø By the break down of brown fat

Ø NEITHER CAN NEWBORN

SHIVER NOR THEY CAN SWEAT

AT BIRTH
• Brown fat: nape of neck, between scapulae, kidneys, adrenals,
axillae, subcutaneous tissue

• Maintain warm chain


Periods of growth
• Ovum:
• Embryo:
• Fetus:
• Perinatal period:
Periods of growth
• Ovum: 0-2 weeks
• Embryo: 3-8 weeks
• Fetus: 8w- birth
• Perinatal period:
• 22 weeks POG – 7days
Ref:Nelson 20/e p b

Q9: Embryonic period is up to ?

(Recent quest 2019)

A. 6 weeks

B. 8 weeks

C. 10 weeks

D. 12 weeks
• What is the normal pupil of a newborn-

• Dilated or constricted ?
Pupil of a newborn are small & difficult to

dilate, due to immature IRIS DILATOR

muscle, therefore it Is constricted !


Q11: The pupil of a newborn is:

FMGE NOV 2017

A. Dilated

B. Mid dilated

C. Constricted

D. Normal

Ref: Nelson 20th e p c


•HARD WORK WILL PAY !!
APGAR Score
>7 Normal
4-7 Moderate birth asphyxia

>7 Normal
Score 0-3 Severe birth asphyxia

4-7 Moderate birth asphyxia

>7 Normal
Q34: If APGAR score is 6, at 5 min of life, then what is

the interpretation: 2018

A. Child is normal

B. Moderate asphyxia

C. Severe birth asphyxia

D. It require immediate resuscitation

Ref: Nelson 20th e b


Q34: If APGAR score is 6, at 5 min of life, then what is

the interpretation: 2018

A. Child is normal

B. Moderate asphyxia

C. Severe birth asphyxia

D. It require immediate resuscitation

Ref: Nelson 20th e b


Q35: APGAR acronym stands for AIIMS
2017,

A. Activity, Pulse pressure, Grimace


Appearance ,Rate of respiration

B. Appearance, pressure, grimace, MAP,


heart rate

C. Appearance, pressure, grimace,


appearance, heart rate

D. Appearance, pulse, grimace , activity,


respiration
Ref: Nelson 20th e d
Q35: APGAR acronym stands for AIIMS
2017,

A. Activity, Pulse pressure, Grimace


Appearance ,Rate of respiration

B. Appearance, pressure, grimace, MAP,


heart rate

C. Appearance, pressure, grimace,


appearance, heart rate

D. Appearance, pulse, grimace , activity,


respiration
Ref: Nelson 20th e d
• How to make a preparation chart so that you
can remember at the end of exams ?
•Revision chit
•Page Number
•High yield content
Summarizes big table in 1
•Wall Post
•Be Imaginative
There is no short cut to success
There is no short cut to success
We all have taken the stairs
•Neonatal sepsis
Q35:Most common cause of neonatal sepsis

APGAR acronym stands for AIIMS
2017,throughout the world:

A.
➤ In hospitals:
Activity, E.Coli*
Pulse pressure, Grimace
➤ Appearance ,Rate of respiration
Overall: Gr B Streptococcus*

B. ➤ Appearance,
Early Onset: Grpressure, grimace,
B streptococcus* MAP,
heart rate
➤ MOST COMMON CAUSE OF
C. Appearance, pressure,
NEONATAL SEPSIS grimace,
IN INDIA (OR IN
appearance,
HOSPITALS INheart rate
INDIA):

D. ➤ Appearance, pulse,
ACINETOBACTER grimace , BY
FOLLOWED activity,
respiration
KLEBSIELLA*
Ref: Nelson 20th e d
INVESTIGATIONS
➤ BLOOD CULTURE: GOLD STANDARD (Confirmatory)
➤ SEPTIC SCREEN: ANY 2 OR MORE THAN 2

TOTAL LEUCOCYTE COUNT <5000/cumm

Low - AS PER MANROE CHART


ABSOLUTE NEUTROPHIL
FOR TERM AND MOUZINHO’S
COUNT
CHART FOR VLBW

IMMATURE/TOTAL
>0.2
NEUTROPHIL

MICRO ESR >15MM IN FIRST HOUR

CRP >I mg/dl (+ve)


•Life is 1 time offer
•Life is 1 time offer
•Use it well
• MD Pediatric Exam – Indore
• MD Pediatric Exam – Indore
• 7 candidates
• MD Pediatric Exam – Indore
• 7 candidates
• 1 newborn – 30 min
• MD Pediatric Exam – Indore
• 7 candidates
• 1 newborn – 30 min
• +ve findings ?
• MD Pediatric Exam – Indore
• 7 candidates
• 1 newborn – 30 min
• +ve findings ?
• 6 Failed
• MD Pediatric Exam – Indore
• 7 candidates
• 1 newborn – 30 min
• +ve findings ?
• 6 Failed
• 1 Passed
•Healthy Normal Baby
• Conditions in a newborn which looks
abnormal but are normal?
• Physiological weight loss
• Physiological weight loss
•T
•T
•T
• Physiological weight loss
• Term
• Ten
• Tenth
•Hymenal tag ?
Q. A hymenal tag in a newborn is best treated by:

RECENT QUESTION 2016

MAY

A. Steroids

B. Surgery

C. Leaving at alone

D. None of the above

Ref: Nelson 20/e p 63, Ghai 8/e p 134-136c


Q. A hymenal tag in a newborn is best treated by:

RECENT QUESTION 2016

MAY

A. Steroids

B. Surgery

C. Leaving at alone

D. None of the above

Ref: Nelson 20/e p 63, Ghai 8/e p 134-136c


• Black neonates, contains neutrophils

• Site: chin, neck, back-lasts for 2-3 days


• Milia: benign, no treatment required, self resolving,

nose, chin, cheeks, keratin becomes trapped

beneath the skin.


Epstein pearls: epithelial inclusion cyst mainly on

hard palate
• Harlequin color change:
15.Harlequin skin change is seen due to
mutation of which gene?
a. ABCA 12
b. FAD
c. Keratin 1
d. ALOXE 3
15.Harlequin skin change is seen due to
mutation of which gene?
a. ABCA 12
b. FAD
c. Keratin 1
d. ALOXE 3
Conditions in a newborn

which looks abnormal but

are normal?

v Erythema toxicum:

v Mongolian spots:
vStork bite/ salmon

patch/ nevus simplex:

vMilia:

vEpstein pearls:

vAcne neonatorum:
Others:
• Others:

• Subconjunctival hemorrhage

• Bilateral breast enlargement

• Vaginal bleeding

• Physiological weight loss

• Hymenal tags

• Physiological phimosis

• Galactorrhea of newborn
Q: Gynecomastia in a neonate is seen due to:

FMGE NOV 2017

A. Mother estrogen

B. Mother progesterone

C. GnRH

D. gonadotropins

Ref: Nelson 20/e a p a


Q: Gynecomastia in a neonate is seen due to:

FMGE NOV 2017

A. Mother estrogen

B. Mother progesterone

C. GnRH

D. gonadotropins

Ref: Nelson 20/e a p a


Q14: True about
cephalohematoma is/are:

JAN 2018

A. Limited by sutures

B. Cause due to periosteal injury

C. Due to fluid accumulation

D. Regresses in 24 hours

E. It is due to birth trauma

Ref: NELSON 20/ e abe


Q14: True about
cephalohematoma is/are:

JAN 2018

A. Limited by sutures

B. Cause due to periosteal injury

C. Due to fluid accumulation

D. Regresses in 24 hours

E. It is due to birth trauma

Ref: NELSON 20/ e abe


Ref: Nelson 20/e p b

Q20: About caput succedaneum all are


true EXCEPT:

Recent ques 2012

A. Soft

B. Prolonged jaundice

C. Ill defined

D. None are true


Ref: Nelson 20/e p b

Q20: About caput succedaneum all are


true EXCEPT:

Recent ques 2012

A. Soft

B. Prolonged jaundice

C. Ill defined

D. None are true


Normal temp. of a newborn ?
• Mild

• Moderate

• Severe

• Axillary temp – duration ?


Q24: A newborn looses maximum heat from: Ref: Nelson 20th e ap 2798, ghai 126

FMGE 2018

A. Head

B. Abdomen

C. Palms and soles

D. Neck
This little stress only will make you shine
vYour future depends on what you
do today
vYour future depends on what you
do today

& your today has already started..


• Neonatal Resuscitation:
• With New Updates
Initial steps
• Baby crying?

• Routine care

• Baby not crying?

• Initial steps PSSR

• SUCTION MOUTH FIRST THEN NOSE

• Size of suction catheter: 12 to 14 F

• Suction pressure: 80mm Hg or 100

cm H20
•Baby still not crying?
• Check HR

• HR > 100:

• HR < 100:

• HR < 60:
• HR > 100 : PSSR + spo2 monitor, supplementary

o2 / CPAP

• HR < 100 : BMV/ spo2 monitor / ECG MONITOR

• HR < 60 - BMV + CC,

• 3:1

• 1/3rd chest
If second time heart rate < 100 bpm then ?
If second time heart rate < 100 bpm then

Ventilation corrective steps


Q: Ventilation corrective steps ?
Q: Ventilation corrective steps ?
• HR < 60 - BMV + CC + drugs

• 3:1

• 1/3rd chest
• Indication of giving adrenaline

• HR < 60 after 1 minutes of CC + BMV

• Adr is given via IV route


Q: Which ET tube you will use
for 900 grams baby ?
Q: Which ET tube you will use
for 900 grams baby ?
What is the Spo2 at first
& 5* minute of life ?

AIIMS
What is the Spo2 at
first & 5 minute of life
?
Indication
Contraindication
Technique & Rate
Self inflating bag
240-750 ml capacity

v HR < 100 bpm, apnea, gasping

v Contraindication: CDH *

v E C clamp technique

• Rate: 40-60 cpm

• Recommended pressure:

• 1st breath: 30-40 cm H20

• SUBSEQUENT BREATH: 15-20 CM H20


B 25 cm H2o
HR < 60 : BMV + CC, 3:1, 1/3rd chest
B

A
Which statement is correct ?

• Method a is preferred over method b

• Method b is preferred

• Both method are good

• Method b is better when there is one resuscitator only

AIIMS MAY 2015


Which statement is correct ?

• Method a is preferred over method b

• Method b is preferred

• Both method are good

• Method b is better when there is one resuscitator only

AIIMS MAY 2015


Important points

• Most important INDICATOR for successful resuscitation

• Most important INDICATOR for successful ventilation


Important points

• Most important INDICATOR for successful

resuscitation: VENTILATION

• Most important INDICATOR for successful ventilation:

RISING HEART RATE


Important points
• 10% newborns require some assistance at

birth

• <1 % require chest compressions and

medications

• 60 seconds (the golden minute)* are allotted

for completing initial steps, re evaluating and


beginning ventilation if required

• Ventilation* is the most important step for


successful resuscitation
• Most important indicator
of successful PPV is ?

• Preferred method of
obtaining emergency
vascular access in
delivery room is ?

• What is vigorous and


non vigorous meconium
stained baby?
• Rising heart rate* A rise in heart rate is the most
important indicator of effective ventilation and
response to resuscitative interventions.

• UMBILICAL VENOUS CATHETER

• HR > 100, good muscle tone, good efforts

• Non vigorous newborns with MSL DO NOT REQUIRE


TRACHEAL INTUBATION AND TRACHEAL SUCTIONING
• MOST COMMONEST
CAUSE OF DISTRESS IN

• PRETERM

• TERM

• POST TERM
• Fluid in the interlobar / horizontal fissure
• Most important
indicator of successful
PPV is ?

• Rising heart rate


• How to overcome failures in life ?
•Failure of tactics
•Failure of tactics
•Failure of strategy
•Failure of tactics
•Failure of strategy
•Failure of vision
NEONATAL
HYPERBILIRUBINEMIA
Who will tell me the bilirubin
metabolism ?
Bilirubin uridine diphosphate

glucuronosyl transferase (bilirubin-

UGT) enzyme
Gilbert: G: Good type
C: Cancelled UGT
Crigler: D: Dark liver
R: Rark liver
Dubin:

Rotor:
G: Good type
vGilbert: Immature UGT enzyme (inc.
C: Cancelled UGT
unconjugated bilirubin) D: Dark liver
R: Rark liver
vPt is usually asymptomatic unless they

do exercise, stress.
vCrigler Najjar: absent UGT(Inc unconjugated bilirubin )
Crigler type 1: deficient UGT

Crigler type 2: milder form of deficiency of UGT

Rx of Type 2: Barbiturates ( they inc UGT enzyme)


v Dubin johnson: enzyme is very well functioning but there is un

excretable bilirubin ( inc conjugated bilirubin) , dark liver

appearance
Rotor: mild form of dubin, mildly un excretable jaundice
Ref: Nelson 20th e a b

Q126: Conjugated hyperbilirubinemia is/are seen in:

Recent NOV 2015

A. Rotor’s syndrome

B. Dubbin Johnson syndrome

C. Gilbert’s syndrome

D. Breast milk jaundice

E. Crigler Najjar syndrome


Ref: Nelson 20th e a b

Q126: Conjugated hyperbilirubinemia is/are seen in:

Recent NOV 2015

A. Rotor’s syndrome

B. Dubbin Johnson syndrome

C. Gilbert’s syndrome

D. Breast milk jaundice

E. Crigler Najjar syndrome


A. Gilbert syndrome: 1. Complete unextractable bilirubin

B. Crigler Najjar syndrome: 2. Partial unextractable bilirubin

C. Rotor syndrome : 3. Immature UGT

D. Dubin syndrome : 4. Absent UGT

Q: Match the following:


AIIMS/FMGE NEW PATTERN
A: c

B: d

C: b

D: a
PHYSIOLOGICAL PATHOLOGICAL
PHYSIOLOGICAL PATHOLOGICAL

NEVER APPEAR IN MAY APPEAR


1ST 24 HOURS

DOES NOT STAIN MAY STAIN


PALM AND SOLES

URINE DOES NOT HIGH COLOURED


STAIN DIAPER URINE AND CLAY
COLORED STOOL

DOES NOT PERSIST MAY PERSIST


BEYOND 3 WEEKS BEYOND 3 WEEKS
A newborn produces how much
bilirubin per day ?
A newborn produces how much
bilirubin per day ?

6-10mg of bilirubin per kg per day


1 gm of hemoglobin produces _____ of

bilirubin. ?
1 gm of hemoglobin produces _34mg___ of

bilirubin. ?
To make the jaundice evident – how much

mg/dl should be the bilirubin ?

A. 4

B. 5

C. 6

D. 7
To make the jaundice evident – how much

mg/dl should be the bilirubin ?

A. 4

B. 5

C. 6

D. 7
Q133: 1 gram of HB liberate how many mg of bilirubin:

Recent ques 2013

A. 40

B. 34

C. 15

D. 55
PHYSIOLOGICAL PATHOLOGICAL
Physiological jaundice in a full term newborn usually resolved by
____ weeks
Physiological jaundice in a full term newborn usually resolved by 2
weeks *
PHYSIOLOGICAL PATHOLOGICAL

NEVER APPEAR IN MAY APPEAR


1ST 24 HOURS

DOES NOT STAIN MAY STAIN


PALM AND SOLES

URINE DOES NOT HIGH COLOURED


STAIN DIAPER URINE AND CLAY
COLORED STOOL

DOES NOT PERSIST MAY PERSIST


BEYOND 3 WEEKS BEYOND 3 WEEKS
Breast feeding jaundice:

• Breast milk jaundice:


Breast feeding jaundice: early onset
Breast milk jaundice - late onset
• Unusual metabolite: 3 alpha 20 beta diol
Ref: Nelson 20th e A

Q116: A 4 day old breast fed, otherwise well, term

neonate presents with jaundice, on testing the bilirubin

level was found to be 18mg/dl. Which of the following

is the best step of management?

NOV 2017

A. Start phototherapy & continue breast feeding

B. Start IV fluids & given phototherapy

C. Initiate exchange transfusion

D. Stop breastfeeding & do phototherapy


Ref: Nelson 20th ed

Q123: Cause of jaundice in day 1 baby are all EXCEPT:

Recent ques 2016

A. Rh incompatibility

B. ABO incompatibility

C. Prematurity

D. Breast milk jaundice


Ref: Nelson 20th ed

Q123: Cause of jaundice in day 1 baby are all EXCEPT:

Recent ques 2016

A. Rh incompatibility

B. ABO incompatibility

C. Prematurity

D. Breast milk jaundice


qMost common cause of jaundice on day 1

is:
qMost common cause of jaundice on day 1 is:

erythroblastosis fetalis*
qJaundice appears in ______way ?
qJaundice appears in a cephalo caudal way
KRAMMER’S RULE
1.4-6mg/dl

2.6-8

3.8-12

4.12-14

5.>15
KRAMMER’S RULE
Rx: options of JAUNDICE ??
Rx: Phototherapy

Wavelength:

Mechanism:

Cover the ?
Rx: Phototherapy

450-460 nm/ 460-490nm, LED lamps

Mechanism: photo isomerization: it converts

toxic(insoluble) form of bilirubin into non

toxic
Structural isomerization*** (most

imp): bilirubin converts into

LUMIRUBIN which is easily

excreted

Photo oxidation

Cover the genitals, eye during


phototherapy
Any syndrome it causes ?
Bronze baby syndrome
Dark, grayish brown discoloration
• Gray baby syndrome

• Red man syndrome

• Blue baby syndrome


• Gray baby syndrome - chloramphenicol

• Red man syndrome - vancomycin

• Blue baby syndrome - amiodorone


Which is the most common hemolytic

disease in a newborn ?
Mother: O & baby other than O

PS: micro spherocytes *, reticulocytosis


Q: The item shown below is used to feed newborns at
what gestation?

• 26 weeks
• 27 weeks

• 33 weeks
• 35 weeks
If you are not willing to work
for it
Don’t complain about not
having it
Feeding of preterm neonates:
Feeding of preterm neonates:

• < 28 weeks

• 28-31 weeks

• 32-34 weeks

• > 34 weeks
Feeding of preterm neonates:

• < 28 weeks – inadequate sucking efforts -

• 28-31 weeks

• 32-34 weeks

• > 34 weeks
Feeding of preterm neonates:

• < 28 weeks – inadequate sucking efforts – IV Fluids

• 28-31 weeks

• 32-34 weeks

• > 34 weeks
Feeding of preterm neonates:

• < 28 weeks – inadequate sucking efforts – IV Fluids

• 28-31 weeks – sucking bursts – lack of coordination between sucking/Swallowing & breathing

• 32-34 weeks –

• > 34 weeks
Feeding of preterm neonates:

• < 28 weeks – inadequate sucking efforts – IV Fluids

• 28-31 weeks – sucking bursts – lack of coordination between sucking/Swallowing & breathing

• 32-34 weeks – coordination b/w breathing & swallowing begins

• > 34 weeks
Feeding of preterm neonates:

• < 28 weeks – inadequate sucking efforts – IV Fluids

• 28-31 weeks – sucking bursts – lack of coordination between sucking/Swallowing & breathing

• 32-34 weeks – coordination b/w breathing & swallowing begins

• > 34 weeks – mature sucking patterns


Q: The item shown below is used to feed newborns at
what gestation?

• 26 weeks
• 27 weeks

• 33 weeks
• 35 weeks
Q: The item shown below is used to feed newborns at
what gestation?

• 26 weeks
• 27 weeks

• 33 weeks
• 35 weeks
Q45: A 32 week, 1400gms neonate is born to primigravida. The
baby did not require resuscitation and show stable vitals. The
baby was transferred to the NICU. How will you manage the
feeding of the patient?

AIIMS MAY 2016

A. Start total enteral feeding & IV nutrition not required

B. Start IV nutrition with minimal enteral feeding

C. Start IV nutrition and introduce feeds on 2nd day of life

D. Start parenteral nutrition and institute oral feeding on 2nd


day of life

Ref: Nelson 20 th e p 790, Ghai 8 th e p 158a


Q45: A 32 week, 1400gms neonate is born to primigravida. The
baby did not require resuscitation and show stable vitals. The
baby was transferred to the NICU. How will you manage the
feeding of the patient?

AIIMS MAY 2016

A. Start total enteral feeding & IV nutrition not required

B. Start IV nutrition with minimal enteral feeding

C. Start IV nutrition and introduce feeds on 2nd day of life

D. Start parenteral nutrition and institute oral feeding on 2nd


day of life

Ref: Nelson 20 th e p 790, Ghai 8 th e p 158a


Reflex?
• SYMMETRIC Abduction & extension of upper
limbs with

• Opening of hands followed by flexion and


adduction

• Baby cry
• SEQUENCE of appearance:
• Opening of hands (28 weeks)*

• Abduction and extension (32 weeks )

• Adduction and flexion ( 37 weeks)

• Disappearance: 5-6 months *


• Persistence of Moro's:

• Exaggerated Moro’s:

• Absent Moro’s:

• Asymmetric Moro’s:

• Most common bone to


# in a newborn
q Persistence of Moro’s:

CEREBRAL DAMAGE*

q Exaggerated Moro’s: Stage

1 HIE, Hypoglycemia

q Absent Moro’s: Stage 3

HIE, down syndrome(

POOR MORO), kernicterus


• Asymmetric Moro’s: # clavicle,

Erb’s palsy*C5C6** shoulder

joint dislocation, congenital

hemiplegia
Q58: Asymmetric Moro's reflex at birth is indicative of:
Ref: Nelson 20th e p c

AIIMS MAY 2014

A. HIE

B. Brain damage

C. Erb’s palsy

D. Kernicterus
Q58: Asymmetric Moro's reflex at birth is indicative of:
Ref: Nelson 20th e p c

AIIMS MAY 2014

A. HIE

B. Brain damage

C. Erb’s palsy

D. Kernicterus
• Erb’s palsy
• ATNR. Presents at birth dis. By 6mnths**

• So STNR appears after birth


Drug of choice
Spotter..
Neonatal seizures: Phenobarbitone
•GIT
• Acute diarrhea
14. The most common cause of
diarrhea in children is-
a. Vibrio cholera
b. E. coli
c. Rotavirus
d. Pneumococcus
14. The most common cause of
diarrhea in children is-
a. Vibrio cholera
b. E. coli
c. Rotavirus
d. Pneumococcus
Eagerly drinking ?

A. Moderate dehydration
B. Severe dehydration
C. None of the above
D. A+B both are correct
Eagerly drinking ?

A. Moderate dehydration
B. Severe dehydration
C. None of the above
D. A+B both are correct
Eagerly drinking + absent tears

A. Moderate dehydration
B. Severe dehydration
C. None of the above
D. A+B both are correct
Eagerly drinking + absent tears

A. Moderate dehydration
B. Severe dehydration
C. None of the above
D. A+B both are correct
Eagerly drinking + absent tears + dec urine
output ?

A. Moderate dehydration
B. Severe dehydration
C. None of the above
D. A+B both are correct
Eagerly drinking + absent tears + dec urine
output ?

A. Moderate dehydration
B. Severe dehydration
C. None of the above
D. A+B both are correct
Eagerly drinking + absent tears + dec urine
output + refusal to feed

A. Moderate dehydration
B. Severe dehydration
C. None of the above
D. A+B both are correct
Eagerly drinking + absent tears + dec urine
output + refusal to feed

A. Moderate dehydration
B. Severe dehydration
C. None of the above
D. A+B both are correct
•Absent tears + dec urine
output + skin turgor goes
slowly
•It can happen in moderate as
well as severre dehydration
• Absent tears + dec urine output + skin
turgor goes slowly
• It can happen in moderate as well as
severre dehydration
• Refusal to feed = severe dehydration
• Remain calm & focused
•\
Sab theek hojayega !
• Maintenance : holiday & segar formula

• 1-10 kg

• 10-20 kg

• >20 kg
• Maintenance : holiday & segar formula

• 1-10 kg: 4 ml per kg per hr

• 10-20 kg

• >20 kg
• Maintenance : holiday & segar formula

• 1-10 kg: 4 ml per kg per hr

• 10-20 kg: 40 ml + 2 ml per kg hr

• >20 kg
• Maintenance : holiday & segar formula

• 1-10 kg: 4 ml per kg per hr

• 10-20 kg: 40 ml + 2 ml per kg per hr

• >20 kg: 60 ml + 1 ml per kg per hr


13. Most common cause of shock in
child-
a. Septic shock
b. Hypovolemic shock
c. Cardiogenic shock
d. Anaphylactic shock
13. Most common cause of shock in
child-
a. Septic shock
b. Hypovolemic shock
c. Cardiogenic shock
d. Anaphylactic shock
• __________ is the most common cause of shock
in children.
• The 2nd most common cause — ____

• 3rd most common — ___________


• Hypovolemia is the most common cause of
shock in children.
• The 2nd most common cause — Septic or
distributive shock.
• 3rd most common — Cardiogenic shock
•Take risks in your life
•Take risks in your life

•If you win you can LEAD !!


•Take risks in your life

•If you win you can LEAD !!

•If you loose

•You can GUIDE !!


• Pediatric Cardiology
• Pediatric Cardiology
• Most common
• Pediatric Cardiology
• Syndrome specific CHD
•Trilogy

•Tetralogy

•Pentalogy
•Trilogy

•Tetralogy: ? PS

•Pentalogy
In Fetal circulation

• Left to right shunt / right to left shunt


• Right to left – polycythemia / brain abscess
In Fetal circulation

• Oligemic lung fields ?

• Plethoric lung fields ?


• Fetal circulation
Structure Remnant

Ductus arteriosus a)Lig arteriosum


b)Lig venosum

Umbilical artery

Umbilical vein

Ductus venosus
Structure Remnant

Ductus arteriosus Ligamentum


func 10-15 hrs, anat. arteriosum
10-21d,
Umbilical artery Medial umbilical
ligament

Umbilical vein Ligamentum teres


hepatis

Ductus venosus Ligamentum


venosum
In Fetal circulation
• Ductus Venosus is a bypass shunt between ____________?

A. SVC & IVC

B. PORTAL VEIN & IVC

C. RIGHT SPLEENIC VEIN & PORTAL VEIN

D. PORTAL VEIN & SVC


In Fetal circulation
• Ductus Venosus is a bypass shunt between ____________?

A. SVC & IVC

B. PORTAL VEIN & IVC

C. RIGHT SPLEENIC VEIN & PORTAL VEIN

D. PORTAL VEIN & SVC


• During fetal circulation: PVR _____. and SVR ____.

A. Increases & decreases


B. Decreases & decreases
C. only increases
D. Only decreases
• During fetal circulation: PVR _____. and SVR ____.

A. Increases & decreases


B. Decreases & decreases
C. only increases
D. Only decreases
In Fetal circulation

• In adults gas exchange occur in ____, and in


fetus its through ____

A. placenta / lungs

B. lungs / placenta

C. both lungs & placenta

D. none of the above


In Fetal circulation

• In adults gas exchange occur in ____, and in


fetus its through ____

A. placenta / lungs

B. lungs / placenta

C. both lungs & placenta

D. none of the above


In Fetal circulation

• In fetus lungs are ______.

A.Hyperinflated

B.Collapse

C.Working normally

D.Partially working
In Fetal circulation

• In fetus lungs are ______.

A.Hyperinflated

B.Collapse

C.Working normally

D.Partially working
• How will the patient present ?
• How will the patient present ?
Egg On String - TGA
What is true regarding TGA ?

A. It’s a duct independent congenital heart

B. It is the most common acyanotic congenital heart disease

C. Parallel circulation

D. On X ray we will see figure of 3 appearance


What is true regarding TGA ?

A. It’s a duct independent congenital heart

B. It is the most common acyanotic congenital heart disease

C. Parallel circulation

D. On X ray we will see figure of 3 appearance


• How will the patient present ?
• How will the patient present ?
Water bottle shape heart
Water bottle shape heart: pericardial effusion
•Finish
• DDs of Anasarca:
• DDs of Anasarca:
• Anemia

• Malnutrition

• CCF

• Nephrotic syndrome

• Portal hypertension
• Congestive cardiac failure: Causes

• Children
• Congestive cardiac failure: Causes

• Children:
• Rheumatic fever ( beyond 5 yr)
• Congestive cardiac failure: Causes

• Children:
• Rheumatic fever ( beyond 5 yr)
• Rheumatic heart disease
• Congestive cardiac failure: Causes

• Children:
• Rheumatic fever ( beyond 5 yr)
• Rheumatic heart disease
• Systemic hypertension
• Congestive cardiac failure: Causes

• Children:
• Rheumatic fever ( beyond 5 yr)
• Rheumatic heart disease
• Systemic hypertension
• Myocarditis
• Features of right heart failure
• Features of right heart failure
• Peri orbital swelling
• Features of right heart failure
• Peri orbital swelling
• Facial puffiness
• Features of right heart failure
• Peri orbital swelling
• Facial puffiness
• Raised JVP
• Features of right heart failure
• Peri orbital swelling
• Facial puffiness
• Raised JVP
• Tender hepatomegaly
• Features of right heart failure
• Peri orbital swelling
• Facial puffiness
• Raised JVP
• Tender hepatomegaly
• Ascites/ effusion
• Features of right heart failure
• Peri orbital swelling
• Facial puffiness
• Raised JVP
• Tender hepatomegaly
• Ascites/ effusion
• Scrotal edema
• Bilateral pitting edema
• Features of Left heart failure
• Features of Left heart failure
• Tachycardia
• Features of Left heart failure
• Tachycardia
• Tachypnea
• Features of Left heart failure
• Tachycardia
• Tachypnea
• Cough
• Features of Left heart failure
• Tachycardia
• Tachypnea
• Cough
• Wheezing
• Features of Left heart failure
• Tachycardia
• Tachypnea
• Cough
• Wheezing
• Rales in chest
• Features of either side
• Features of either side
• Cardiac enlargement
• Features of either side
• Cardiac enlargement
• Peripheral cyanosis
• Features of either side
• Cardiac enlargement
• Peripheral cyanosis
• Not gaining weight
• Features of either side
• Cardiac enlargement
• Peripheral cyanosis
• Not gaining weight
• Small volume pulse
• Treatment
Grow through What You go through
• Indomethacin and ibuprofen are not effective in _______children ?

A. Preterm

B. Term

C. Late preterm

D. Very preterm

E. Extremely preterm
• Indomethacin and ibuprofen are not effective in _______children ?

A. Preterm

B. Term

C. Late preterm

D. Very preterm

E. Extremely preterm
• Which criteria do you use to diagnose presence of
congenital heart disease ? *
•NADA’S CRITERIA
MAJOR MINOR
MAJOR MINOR

Systolic murmur
grade 3 or more

Any diastolic
murmur
MAJOR MINOR

Systolic murmur
grade 3 or more

Any diastolic
murmur

Cyanosis

Congestive heart
failure
MAJOR MINOR A. 1+2
B. 2+2
Systolic murmur <3 C. 2+1
grade 3 or more D. 1+1

Any diastolic
murmur

Cyanosis

Congestive heart
failure
MAJOR MINOR

Systolic murmur <3


grade 3 or more

Any diastolic Abnormal S2


murmur

Cyanosis

Congestive heart
failure
MAJOR MINOR

Systolic murmur <3


grade 3 or more

Any diastolic Abnormal S2


murmur

Cyanosis Abnormal ECG

Congestive heart Abnormal Chest x


failure ray, BP
MAJOR MINOR

Systolic murmur <3 A. 1+2


grade 3 or more B. 2+2
C. 2+1
Any diastolic Abnormal S2 D. 1+1
murmur

Cyanosis Abnormal ECG

Congestive heart Abnormal Chest x


failure ray, BP
MAJOR MINOR

Systolic murmur <3 A. 1+2


grade 3 or more B. 2+2
C. 2+1
Any diastolic Abnormal S2 D. 1+1
murmur

Cyanosis Abnormal ECG

Congestive heart Abnormal Chest x


failure ray, BP
Chapter:
Respiratory System
Chapter:
Pneumonia
Q: Identify the pathology: Recent ques
2019

A. Congenital diaphragmatic hernia

B. Cong lobar emphysema

C. CCAM

D. Pulmonary sequestration
Hallmark of diaphragmatic hernia

• Scaphoid abdomen

• Contraindicated BMV

• Distress

• Causes compression of lungs

• Left side is common

• Chest: bowel sounds +

• BOCHADALEK HERNIA ( POSTEROLAT )


Q1: Congenital lobar emphysema is commonly
seen in which lobe?
JIPMER MAY 2017
A. Left lower
B. Right upper
C. Left upper
D. Right lower

Ref: Nelson 20/ecp 2113-2116


Q1: Congenital lobar emphysema is commonly
seen in which lobe?
JIPMER MAY 2017
A. Left lower
B. Right upper
C. Left upper
D. Right lower

Ref: Nelson 20/ecp 2113-2116


• How to overcome failures in life ?
•Failure of tactics
•Failure of tactics
•Failure of strategy
•Failure of tactics
•Failure of strategy
•Failure of vision
TEF
• Name of the classification in ?

• Silence classification

• West haven classification

• Gross classification
Esophageal atresia
without TEF
2nd most common
No gas in abdomen
Esophageal atresia
without TEF
2nd most common
No gas in abdomen

Proximal TEF
No gas in abdomen
Esophageal atresia
without TEF
2nd most common
No gas in abdomen Proximal esophageal atresia
with distal TEF

Proximal TEF
No gas in abdomen

Most Common Type


Gas In Abdomen +
Esophageal atresia
without TEF
2nd most common
No gas in abdomen Proximal esophageal atresia
with distal TEF

Proximal TEF
No gas in abdomen

Proximal &
Distal TEF
Least common
Gas in abdomen +

Most Common Type


Gas In Abdomen +
Esophageal atresia
without TEF Isolated TEF
2nd most common (H/n type)
No gas in abdomen Proximal esophageal atresia
with distal TEF Distal gas present

Proximal TEF
No gas in abdomen

Proximal &
Distal TEF
Least common
Gas in abdomen +

Most Common Type


Gas In Abdomen +
Diagnosis:
Diagnosis: Inability to pass NG tube

X-ray
Diagnosis: Inability to pass NG tube

X-ray: Coiled tube in atretic end & gas in the abdomen type C,D & E*

GasLESS abdomen in type A+ B

Rx:
Diagnosis: Inability to pass NG tube

X-ray: Coiled tube in atretic end and gas in the abdomen type C,D & E

Rx: distance b/w both ends <3cm: surgical repair & end to end anastomosis

of esophagus
Diagnosis: Inability to pass NG tube

X-ray: Coiled tube in atretic end and gas in the abdomen type C,D & E

Rx: distance b/w both ends <3cm: surgical repair & end to end anastomosis

of esophagus

If the gap is more: then this is a bad prognostic criteria for surgery **

gastrostomy is done & esophageal repair by conduits (stomach, jejunum)


• Complications:
• Complications: anastomosis leak: Earliest complication *
• Complications: anastomosis leak: Earliest complication *

• Stricture formation: most common complication *


•Work hard

•Success will make noise 1 day


• In choanal atresia, NG tube will not go only,
In TEF coiling will be seen
• If you pass an OG tube, it will go easily in
choanal atresia but not in TEF
•There is no short cut to success
•We all have taken stairs
• MC congenital anomaly of nose is ?
• MC congenital anomaly of nose is CHOANAL
ATRESIA
• In choanal atresia, distress improves while
crying
•kartagener Syndrome
•Where is the defect ?
v Defects in the action of cilia lining

the respiratory tract (lower and


upper, sinuses, Eustachian tube, middle
ear), fallopian tube,
v Normally: Cilia moves the mucus towards

the throat
v Impairment can result in poor mucociliary

clearance
v Leads to repeated URTI, LRTI, bronchiectasis,

middle ear infection


v Swallowing does not required cilia- so there

will be no problem in DYSPHAGIA


• Situs inversus(only heart) totalis (abdominal organs)
•Triad :
•Triad :
•Chronic sinusitis
•Bronchiectasis
•Situs inversus
•Young syndrome Triad ?

•Chronic sinusitis
•Bronchiectasis
•Situs inversus
•Young syndrome Triad ?

•Chronic sinusitis
•Bronchiectasis
•AZOOSPERMIA
• Identify the sound?
What is stridor?
• Abnormal inspiratory, low pitched &
exacerbated by crying, feeding.
• It is an _____ respiratory tract problem

• Narrow or obstruction at the upper


airway tract
• It is an upper respiratory tract problem

• Narrow or obstruction at the upper


airway tract
•Causes of stridor:
• Causes of stridor:
LARYNGEAL OBSTRUCTION

• Laryngomalacia
• MC cause of congenital laryngeal

anomaly is ______?
Laryngomalacia

• MC cause of congenital laryngeal

anomaly is laryngomalacia
Laryngomalacia

• Most common cause of stridor in

infants and children is ?


• Most common cause of stridor in infants

and children in laryngomalacia


Laryngomalacia

• Stridor improves in _______

• Diagnosis: _______

• Rx: _______
Laryngomalacia

• Stridor improves in prone position

• Diagnosis: flexible laryngoscopy

• Rx: observation, symptoms resolves

spontaneously.
Q11: A 10 month old child presents with an inspiratory sound, mother
complaints that the sound increases with cry & decreases when the child
sleeps in prone position. What should be the probable diagnosis?

JIPMER MAY 2016

A. Croup

B. Acute epiglottitis

C. Laryngitis

D. Laryngomalacia

Ref: Nelson 20th edp 2036


Q11: A 10 month old child presents with an inspiratory sound, mother
complaints that the sound increases with cry & decreases when the child
sleeps in prone position. What should be the probable diagnosis?

JIPMER MAY 2016

A. Croup

B. Acute epiglottitis

C. Laryngitis

D. Laryngomalacia

Ref: Nelson 20th edp 2036


Hope is one of those things in life
you cannot live without..
• Most common cause:

• Common cold:

• Pharyngitis:

• Croup:

• Whooping cough:

• Bronchiolitis:
• Most common cause:

• Common cold: RHINOVIRUS

• Pharyngitis:

• Croup:

• Whooping cough:

• Bronchiolitis:
• Most common cause:

• Common cold: RHINOVIRUS

• Pharyngitis: ADENO, STREPTO PYOGENS

• Croup:

• Whooping cough:

• Bronchiolitis:
• Most common cause:

• Common cold: RHINOVIRUS

• Pharyngitis: ADENO, STREPTO PYOGENS

• Croup: PARAINFLUENZA VIRUS

• Whooping cough:

• Bronchiolitis:
• Most common cause:

• Common cold: RHINOVIRUS

• Pharyngitis: ADENO, STREPTO PYOGENS

• Croup: PARAINFLUENZA VIRUS

• Whooping cough: B.PERTUSSIS

• Bronchiolitis:
• Most common viral cause:

• Common cold: RHINOVIRUS

• Pharyngitis: ADENO, STREPTO PYOGENS(bact)

• Croup: PARAINFLUENZA VIRUS

• Whooping cough: B.PERTUSSIS

• Bronchiolitis: (RSV) RESPIRATORY SYNCITIAL VIRUS


• Pneumonia

• Severe pneumonia

• Very severe pneumonia


• Each and every obstacle you
face helps you to grow that
little bit more into the person
you were born to become
Bronchiolitis:

• Cause

• Inv:

• Rx:
Bronchiolitis:

• Cause: RSV(most common)

• Inv:

• Rx:
Bronchiolitis:

• Inv: X-ray: hyperinflation & infiltrates

• Rx:
vBronchiolitis:
v Rx: Symptomatic, antibiotic have ____ ?
vBronchiolitis:
v Rx: Symptomatic, antibiotic have NO ROLE?
vBronchiolitis:
v Main stay of rx: MOIST o2 Inhalation
vBronchiolitis:
v Beta adrenergic & ipratropium are

RECOMMEND OR NOT ?
vBronchiolitis:
v Beta adrenergic & ipratropium are NOT

recommended <6 months


vBronchiolitis:
v Ribavirin- antiviral ( it shortens the course of

illness in HIV pt) **

v RSV Ig & palivizumab*: should be considered


for infants with chronic lung disease,
prematurity.
You did not come this far to go this far
Chapter:
Neurology
Types of NTDs
Types of NTDs
vSpina bifida occulta: OCCULTA MEANS

HIDDEN

vMeningocele

vEncephalocele

vMyelomeningocele
vAnencephaly

vCaudal regression syndrome

vDermal sinus

vTethered cord

vSyringomyelia

vIniencephaly
Spina Bifida occulta: Midline defect without protrusion of spinal
cord or meninges

Asymptomatic
98
98

Occults means hidden


A meningocele consists of a CSF filled meningeal sac only
(Duramater)

Meninges herniate
Spinal cord is normal

A fluctuant midline mass that might trans illuminate


Meningomyelocele(spina bifida cystica )
Ø Location ?

Ø Protrusion of ?

Ø Risk of developing ?
1. Lumbosacral region*

2. Both meninges & spinal cord protrude into the skin of back
ü Hydrocephalus with type 2 Chiari malformation develops in

80% patients
Q2: Content of meningocele:

Recent ques 2017

A. Duramater

B. Spinal cord

C. Brain matter

D. Cauda equina
Q2: Content of meningocele:

Recent ques 2017

A. Duramater

B. Spinal cord

C. Brain matter

D. Cauda equina
• Which sign is seen in spina bifida ?
• Which sign is seen in spina bifida ?
How to refer the patient ?
• Meningomyelocele is covered

with sterile saline soaked gauze

to prevent it from drying,

trauma and infection*


Q9: a case of meningomyelocele was posted for surgery. Till the pt. is

waiting, the covering of the sac will be protected by gauze soak in:

AIIMS MAY 2013

A. NS

B. Tincture iodine

C. Methylene blue

D. Mercurochrome
Q9: a case of meningomyelocele was posted for surgery. Till the pt. is

waiting, the covering of the sac will be protected by gauze soak in:

AIIMS MAY 2013

A. NS

B. Tincture iodine

C. Methylene blue

D. Mercurochrome
• Meningomyelocele in the

lower sacral region leads to:

bowel and bladder

incontinence
• Encephalocele

• Sac like protrusion of the brain and membrane


• Occipital ENCEPHALOCELE WITH ANENCEPHALY
• Rare

• Brain and spinal cord fully


exposed
Craniorachischisis
Iniencephaly
Anencephaly: a large defect of the calvarium, scalp associated with rudimentary brain
Retroflexion of head, defect in occiput and upper spine & nearly absent
neck
• How do you diagnose NTD ?
• Pre natal screening
• Pre natal screening

• Maternal serum for Alpha

fetoprotein
• Amniotic fluid acetyl

cholinesterase
• Amniotic fluid acetyl

cholinesterase

• 16-18th weeks of POG


• Amniotic fluid acetyl

cholinesterase

• 16-18th weeks of POG

• SPECIFIC MARKER
Q1: Which is the best marker for NTD?
FMGE NOV 2017
A. Alpha fetoprotein
B. HCG
C. Pseudocholinesterase
D. Inhibin A
Q1: Which is the best marker for NTD?
FMGE NOV 2017
A. Alpha fetoprotein
B. HCG
C. Pseudocholinesterase
D. Inhibin A
• Sensitive marker for screening – Alpha feto protein

• Specific – acetylcholine esterase


Prevention of NTD

• What to give ?
• How much to give ?
• All women and child bearing age-

give 0.4 mg (400 microgram) of

folic acid daily


• High risk women ( previously

affected child) give 4 mg (4000

microgram) of folic acid daily,

beginning 1 month before the time

of planned conception
Q6: In a lady with history of birth of a previous child with NTD,

amount of folic acid to be given in microgram as secondary

prophylaxis before next conception is:

AIIMS MAY 2015

1. 40

2. 400

3. 4000

4. 500
Q6: In a lady with history of birth of a previous child with NTD,

amount of folic acid to be given in microgram as secondary

prophylaxis before next conception is:

AIIMS MAY 2015

1. 40

2. 400

3. 4000

4. 500
4 mg/ day 3 months before conception.* IAP Quiz
Now it is 5 mg / 5000 microgram
Q7: folic acid requirement in pregnancy for primary prevention of

NTD is:

TMNPGMEE 2015

1. 200 microgram

2. 400

3. 600

4. 800
Q7: folic acid requirement in pregnancy for primary prevention of

NTD is:

TMNPGMEE 2015

1. 200 microgram

2. 400

3. 600

4. 800
Work hard in silence
HYDROCEPHALUS

Dr.Anand
MD Pediatrics
absorption of CSF.
Impaired circulation & absorption of CSF.
Hydrocephalus
Hydranencephaly
Hydranencephaly

Cerebral hemispheres are absent & the remaining cranial


cavity is filled with CSF
Q32: Following features differentiate hydranencephaly
from hydrocephalus:

Recent quest 2016

A. Hydranencephaly is static while hydrocephalus


continue to increase in size

B. Hydranencephaly does not cause increase in head


size

C. Cerebral cortex is deficient/ hypoplastic in


hydranencephaly

D. All of the above


Ref: Nelson 20th ecp 2817
Q32: Following features differentiate hydranencephaly
from hydrocephalus:

Recent quest 2016

A. Hydranencephaly is static while hydrocephalus


continue to increase in size

B. Hydranencephaly does not cause increase in head


size

C. Cerebral cortex is deficient/ hypoplastic in


hydranencephaly

D. All of the above


Ref: Nelson 20th ecp 2817
•Causes of hydrocephalus ?
• Most Common Cause

• Non communicating is

due to aqueduct of

Sylvius – STENOSIS
• Most Common Cause

• Non communicating is due

to aqueduct of Sylvius –

STENOSIS

• Communicating follows a

subarachnoid hemorrhage
Clinical Features: ?
• Increasing head size
• Widely separated sutures
• Widely separated sutures

• Dilated scalp veins


• Macewen sign/ crack pot sign
• Macewen sign/ crack pot sign produced on percussion of skull in

raised ICT
• Setting sun sign due to impingement of dilated

suprapineal recess on brainstem tectum


•Investigation of choice ?
Trans cranial USG
Trans cranial USG: IVH / seizures
•How to improve retainability
for exams?
•Photographic memory
Turner syndrome Noonan Syndrome

Karyotype 45 X0 Karyotype Normal

Infertile due to streak Fertile but delayed


ovaries puberty

Only Females Males or females

Intelligence Normal Intellectual Disability


Present

Heart: Pulmon.Stenosis

Anti mongoloid slant


•Diagram
•Diagram
•Flow chart
•Diagram
•Flow chart
•Pictures
•Teach someone
Be Humble – We All have A Past !
“X Ray Wrist”
–Dr.Anand MD PEDIATRICS
Age Bone

Newborn

3-9 months

1-13 yrs

12-14 yrs

X ray is compared with Gruelich Pyle Atlas


& Tanner Whitehouse method
Age Bone

Newborn Knee

3-9 months Shouder *

1-13 yrs Hand & wrist

12-14 yrs Elbow & hip

X ray is compared with Gruelich Pyle Atlas


& Tanner Whitehouse method
Ref: Nelson 20th p 1768,ghai 8/B

Q13: Which of the following X ray should be advised


for age determination between 1-3 yr. of age ?

PG11

A. Shoulder

B. Wrist

C. Elbow

D. Iliac bones
Ref: Nelson 20th p 1768,ghai 8/B

Q13: Which of the following X ray should be advised


for age determination between 1-3 yr. of age ?

PG11

A. Shoulder

B. Wrist

C. Elbow

D. Iliac bones
Ref: Nelson 20th p 1768,ghai 8/c

- Q14: At the end of 1 yr the number of carpal bones seen in


skiagram of the hand is :

- COMEDK 2008

A. Nil

B. 1

C. 2

D. 3

E. 4
Ref: Nelson 20th p 1768,ghai 8/c

- Q14: At the end of 1 yr the number of carpal bones seen in


skiagram of the hand is :

- COMEDK 2008

A. Nil

B. 1

C. 2

D. 3

E. 4
•X- ray wrist
finished
•DENTITION
Be Humble – We All have A Past !
1st temporary teeth to appear?

1st permanent teeth to appear?


1st temporary teeth to appear: Lower Central Incisor (6-7m)

1st Permanent Teeth


to appear: 1st Molar (6 yr.)
Lower central incisor
First primary teeth
Define delayed dentition

Causes of natal teeth

Causes of delayed dentition

Causes of super numery

teeth
• Define delayed dentition: No teeth by 13 months

• Causes of natal teeth: PEES

• Causes of delayed dentition: FRIED ChoP

• Causes of super numery teeth: 3cgard


Ø Causes of Natal

Teeth: SEEP

Ø Pierre robin
syndrome

Ø Ellis van creveld


syndrome,

Ø Epidermolysis

bullosa

Ø Soto’s syndrome
Pierre Robin Syndrome
• Micrognathia & difficult BMV

• U shape cleft palate

• Glossoptosis

• Autosomal dominant

• Glossoptosis:
• Downward displacement &
• retraction of tongue
Ellis van creveld syndrome

• Short limb dwarfism


• Dystrophic & friable nails
• Polydactyly
• ASD
Epidermolysis bullosa:

• Very fragile skin which blister


easily with minor trauma
• Long narrow face, high forehead, flushed

cheeks and high pointed chin, macrocephaly


• Causes of delayed dentition:
FRIED CaP
• F: familial

• R: rickets

• I: idiopathic* most common *

• E: endocrine: hypothyroidism*,
hypoparathyroidism, hypopituitarism

• D: down syndrome*

• C: cleidocranial dysplasia

• P: progeria
• Causes of super

Numerary teeth: 3cgard


Causes of super Numerary

teeth: 3cgard

Cleft lip

Cleft palate

Cleidocranial dysplasia

Gardner syndrome
types of teeth ?
• In primary dentition PREMOLAR IS NOT

SEEN

• Primary dentition consist of 20 teeth,

which begins at 6 months of age &

completes by 3 yrs of age


Q42: Delayed eruption is usually considered when there are
no teeth by app. by the age of :

Recent quest 2017

A. 18 months of age

B. 9 months

C. 16 months

D. 13 months

Ref: Nelson 20th p 1768,ghai d


Ref: Nelson 20th p 1768,ghai 8/eb

Q43: Eruption of first molar permanent tooth occurs at the age

of ?

WB PGMEE 2016

A. 4-5yr

B. 6-7yr

C. 7-8yr

D. 8-10
Ref: Nelson 20th p 1768,ghai 8/ea

Q44: Most common location of supernumerary


teeth is between?

Recent quest 2014

A. Incisors

B. Canines

C. Molars

D. Premolars
Ref: Nelson 20th p 1768,ghai 8/ea

Q49: Milk teeth total no, in human being:

Recent ques 2013

A. 20

B. 28

C. 32

D. 24
A. Mulberry molars
Identify? B. Natal teeth
C. Supernumerary teeth
D. Hutchinson’s teeth
Mulberry molars: multiple rounded

Rudimentary enamel cusps

on the permanent first: congenital syphilis


•DENTITION
FINISHED
Stay committed to your decisions but flexible to your approach !!
Q: FIRST VISIBLE SIGN OF PUBERTY IN GIRLS?

NEET JUNE 2020-

A. Thelarche
B. Pubarche
C. Menarche
D. Growth dev
Sequence of
changes in puberty:
Girls
• Thelarche(breast) (8-
12yr)*
• Thelarche(breast) (8-
12yr)*

• Pubarche
• Thelarche(breast) (8-
12yr)*

• Pubarche

• Growth spurt (peak


growth occur in stage
3)*
• Thelarche(breast) (8-
12yr)*

• Pubarche

• Growth spurt (peak


growth occur in stage
3)*

• Menarche(2-2.5yr
later during SMR3-4
Boys
• Testicular enlargement
• Testicular enlargement

• Penile enlargement
• Testicular enlargement

• Penile enlargement

• Pubic hairs
• Testicular enlargement

• Penile enlargement

• Pubic hairs

• Growth spurt
• Testicular enlargement

• Penile enlargement

• Pubic hairs

• Growth spurt

• Axillary hair
• Testicular enlargement

• Penile enlargement

• Pubic hairs

• Growth spurt

• Axillary hair

• Facial hair
JOURNEY OF
MILESTONES
3M

4M

5M
3M-neck holding

4M-no head lag

5M-complete neck

control, feet to

mouth
What is the probable age

of the baby?

A. 2 weeks

B. 6 weeks

C. 8 weeks

D. 12 weeks
Prone position

2 weeks:

high pelvis &

knees drawn

up
Prone position
Prone position

6 weeks: flat

pelvis and

extended

hips
Child rolls over by ?

Recent ques 2016

A. 3 months

B. 6 months

C. 7 months

D. 8 months
• Why role over is important ?
Once the asymmetric tonic neck

reflex disappear,

baby learns to roll over from

prone to supine and vice versa.


SITTING & STANDING
• Sitting with support/TRIPOD : (6-
7) months

• Sitting without support: (8)


months
On the ground

Off the ground


crawl (8)

creeps (10)
• (9) stand holding the furniture (with support)

• (10-11) cruising + pivoting (turn around to pick an

object)

• (12)stands without support


• Independently stand ?
Independently stand ?

Independently walk?
Independently stand ?

Independently walk?

Run?
Independently stand ?

Independently walk?

Run?

And then ?
• Independently stand ?

• 12 months

• Walk with one hand held?

• 12-13 months

• Independently walk?

• 15 months
• Run?

• 18 months

• And then ?

• 2,3,4 yrs. (stairs)


• Copies parents in task? 2p 15m

• Pulls people to show toys/ share toys?

• Know full name and gender?


• Copies parents in task?

• 18 months

• Pulls people to show toys/ share


toys?

• 2 yr, 3 yr.*

• Know full name and gender?

• 3 yr.*
• Plays cooperatively in a
group, goes to toilet
alone?
• Plays cooperatively in a
group, goes to toilet
alone?

• 4 years
2 yr: Stairs
3 yr:

4 yr:

5 yr:
Stairs
• 2 yr: walk backwards, kicks a

ball*, 2 feet at 1 step

• 3 yr: tricycle, alternate feet

• 4 yr: hopping, comes

downstairs

• 5 yr: skipping
•Gross motor
finished
FINE MOTOR
Hand regard
Midline
Mouth
Hand regard- 3
Midline - 4
Mouth - 5
Bidextrous – 4-5 months
Unidextrous – 6-7 months
1 month: hand kept closed FINE MOTOR:
2 month: hand intermittently open

3 month: hand regard appear

4 month: tries to reach out an object

5 month: bidextrous grasp, mouthing

6 month: unidextrous/palmar grasp*, takes


biscuit to mouth

7 months: transfer objects

9 months: immature/pincer grasp*

12 months: mature pincer grasp/ casting*


Dressing/ Undressing
• (3yr) dress and undress completely if
helped with buttons

• 4 yrs
• (3yr) dress and undress completely if
helped with buttons

• 4 yrs button unbutton comes

• (5yr) tie shoelaces as well


Drawing skills
Personal $ social
• Gives regard to the mother ?

• Fixate the eye to the mother ?


• Gives regard to the mother ?

• Fixate the eye to the mother ?

• 1 month
Personal $ social

Define social smile?


Personal $ social
Define social smile?

• Social smile is when baby


starts smiling when anyone
talks to him
Social Smile
Social Smile
2 months

No social smile
by 3 months
Is a
Red flag sign
• Recognizes mother ?

• Stranger anxiety (anxious on


meeting strangers)?
• Recognizes mother – 3 m

• Stranger anxiety 6-7 m


6 month: mirror play
• (9m) bye bye

• (12m) comes when called


Which game is this ?
•10 month: plays peek a boo*
• 15 months: 2p-

• 18 months: 2d- dry by day.

• 36 months: DRY AT NIGHT


Language

• 1 month:
Language
1 month: quietens when a bell is rung
Language
• 2 month:
• 3 months:
Language
• 2 month: vocalizes
• 3 months: cooing
•laughs loud ?
•4 months: laughs loud
• Which mile stone is this ?
•5 m: razzing ?
: Monosyllables
: Bisyllables
•6 months: Monosyllables
•9 months: Bisyllables
• 1st birthday –

• 18 months –

• 2 years –

• 2 years –
• 1st birthday – 1-2 word with meaning

• 18 months – vocab of 8-10 words

• 2 years – 100 words

• 2 years – 2-3 word making


sentences
Vision and hearing
Vision and hearing

• Birth:

• 4 weeks:

• 8-12 weeks(2-3m):

• 3-4 months:

• 6 months:
Vision and hearing
• Birth: regards a dangling object and
follows - 45 degree

• 4 weeks: 90 degree

• 8-12 weeks(2-3m): 180 degree


• 3-4 months: binocular vision
established

• 6 months: visual acuity of an


adult develops
• Hearing:
DEVELOPMENTAL QUOTIENT =

(IQ) Intelligent Quotient =


• DEVELOPMENTAL QUOTIENT =

DEVELOPMENTAL AGE/ ACTUAL AGE

* 100

• (IQ) Intelligent Quotient = mental

age / chronological age * 100


Q: At what years can a
child do the following ?

A. 3

B. 4

C. 5

D. 6

Very important
• What is the cut off level of imbecile ?
• Imbecile = stupid person
IQ
NORMAL 90-110
IMBECILE 20-50

Idiot <20
•What is TORCH profile ?
T-

O–

R-

C-

H-
T- toxoplasmosis

O–

R-

C-

H-
T- toxoplasmosis

O – others (syphilis, varicella, HIV )

R-

C-

H-
T- toxoplasmosis

O – others (syphilis, varicella, HIV )

R- rubella

C-

H-
T- toxoplasmosis

O – others (syphilis, varicella, HIV )

R- rubella

C- cytomegalovirus

H-
T- toxoplasmosis

O – others (syphilis, varicella, HIV )

R- rubella

C- cytomegalovirus

H- herpes
• Which is the most common infection
among TORCH ?
• Which is the most common infection
among TORCH ?

• CMV
•Which one is bacteria
• T- toxoplasmosis

• PROTOZOA
• T- toxoplasmosis

• PROTOZOA

• Risk of fetal infection

increases with each

trimester

• 15 % - 25 % - 60 %
• T- toxoplasmosis

• PROTOZOA

• COMMON IN 3RD trimester


Congenital toxoplasmosis
Congenital toxoplasmosis
• Hydrocephalus
• Intra cranial calcification(diffuse)
• Chorioretinitis
Congenital toxoplasmosis
• Hydrocephalus
• Intra cranial calcification(diffuse)
• Chorioretinitis ** MOST COMMON
• MATERNAL RX WITH

SPIRAMYCIN DURING

PREGNANCY REDUCES THE

RISK OF TRANSMISSION

SIGNIFICANTLY
Q26: Which of the following agents is likely to cause Cerebral
calcification & hydrocephalus in a newborn who's mother has history
of taking spiramycin but was not compliant with therapy:

AI PG

A. Rubella

B. Toxoplasmosis

C. CMV

D. Herpes
Q26: Which of the following agents is likely to cause Cerebral
calcification & hydrocephalus in a newborn who's mother has history
of taking spiramycin but was not compliant with therapy:

AI PG

A. Rubella

B. Toxoplasmosis

C. CMV

D. Herpes
• O – others syphilis
Syphilis
• Early manifestations:

Snuffles*(syphilitic rhinitis)

• Late: Hutchinson triad*


Syphilis
• Hutchinson teeth:

• Interstitial keratitis

• 8th nerve deafness


Mulberry molars: multiple rounded

Rudimentary enamel cusps

on the permanent first


• Recurrent abortions are seen only in ?
• Recurrent abortions are seen only in ?

• Congenital syphilis
• O – others varicella (1st 2nd

trimester)
Congenital varicella syndrome
• Limb hypoplasia Congenital varicella syndrome
• Cicatrical skin lesions
• h/o fever with rash in end of 1st / 2nd
trimester
CONGENITAL RUBELLA SYNDROME /
GREGG SYNDROME
• Which is the most severe form of intrauterine infection ?
CONGENITAL RUBELLA SYNDROME

• Rubella causes most severe

form of intra uterine

infection
CONGENITAL RUBELLA SYNDROME

• < 11 weeks POG – 90% defects


CONGENITAL RUBELLA SYNDROME

• Forchheimer spots*: it is seen in rubella infection –

not in syndrome
CONGENITAL RUBELLA SYNDROME

• ForchheimeR spots*: Red

spots over palate.


• Triad

• C

• C

• D
• Triad
SNHL + nuclear cataract + PDA = Greggs Triad

• C – cong. Heart - PDA

• C - cataract

• D - deafness
• Which is the most common cong. Heart ?

• Least common ?
• Which is the most common cong. Heart ?

• PDA

• Least common ?

• ASD**
• Which is the most common component ?
• Which is the most common component ?

• Deafness
• Will they have micro or macrocephaly ?
• Will they have micro or macrocephaly ?

• Microcephaly
• Ocular:

• Skin lesions:
• Ocular: salt & pepper retinopathy (most common*)

• Skin lesions: blueberry muffin appearance *


Triad of kwashiorkor
Triad in complication of SAM
Be humble
Be humble
we all have a past
NEET RECALL
2022
questions- PART 2
Dr.anand
If you are not willing to work
for it
Dr.anand
Don’t complain about not
having it
Dr.anand

• 6 YEAR OLD

• Dev delay

• ECG: hypsarrthymias

• Infantile spasm
• Down syndrome

• Edward syndrome

• Patau syndrome

• WEST SYNDROME
• Down syndrome

• Edward syndrome

• Patau syndrome

• WEST SYNDROME
qRx:

A. ACTH

B. Ethosuximide

C. Sodium valproate

D. Lamotrigine
qRx:

A. ACTH

B. Ethosuximide

C. Sodium valproate

D. Lamotrigine

E. Vigabatrin ( ?)
q Rx:

A. ACTH

B. Ethosuximide

C. Sodium valproate

D. Lamotrigine

E. Vigabatrin ( tuberous

scleosis)
qQ: 20 hours old baby came with complaints of not passing

urine. What is the next best step in the management ?

A. Wait and watch

B. Investigate

C. Admit the child

D. None of the above


qQ: 20 hours old baby came with complaints of not passing

urine. What is the next best step in the management ?

A. Wait and watch

B. Investigate

C. Admit the child

D. None of the above


qWhat is high risk among the following ?

A. Weight less than 2.5 kg

B. Weight <10th centile

C. Small for gestational age

D. Large for gestational age

E. All of the above


qWhat is high risk among the following ?

A. Weight less than 2.5 kg

B. Weight <10th centile

C. Small for gestational age

D. Large for gestational age

E. All of the above


qWhat is high risk among the following ?

A. Single parent

B. Twin pregnancy

C. Maternal age <16 & >40

D. Maternal disease like diabetes, htn

E. All of the above


qWhat is high risk among the following ?

A. Single parent

B. Twin pregnancy

C. Maternal age <16 & >40

D. Maternal disease like diabetes, htn

E. All of the above


qWhat is high risk among the following ?

A. Breast feeding

B. Artificial feeding
qWhat is high risk among the following ?

A. Breast feeding

B. Artificial feeding
qOn a routine ultrasound examination – multiple fractures

were present in the scan. What is the next best step ?

A. Osteogenesis imperfecta

B. Achondroplasia

C. Cretinism

D. Marfan syndrome
qOn a routine ultrasound examination – multiple fractures

were present in the scan. What is the next best step ?

A. Osteogenesis imperfecta

B. Achondroplasia

C. Cretinism

D. Marfan syndrome
• Which type of OI affects intrauterine period ?

•1

•2–?

•3

•4
• How many types of OI are present ?

•1

• 2 – perinatal lethal

•3

•4
• How many types of OI are present ?

•1

• 2 – stillborn / die in first year

•3

•4
• How many types of OI are present ?

•1

• 2 – intrauterine #

•3

•4
• How many types of OI are present ?

•1

• 2 – intrauterine # / crumpled appearance

•3

•4
• Q: which of the following is autosomal recessive condition
?
• Q: which of the following is autosomal recessive condition
?

A. Achondroplasia

B. Treacher collin syndrome

C. Osteogenesis imperfecta

D. None of the above

E. Cystic fibrosis
• Q: which of the following is autosomal recessive condition
?

A. Achondroplasia

B. Treacher collin syndrome

C. Osteogenesis imperfecta

D. None of the above

E. Cystic fibrosis
AUTOSOMAL DOMINANT INHERITENCE ?
AUTOSOMAL DOMINANT INHERITENCE

T
AUTOSOMAL DOMINANT INHERITENCE

D – Dystrophia Myotonica

T
AUTOSOMAL DOMINANT INHERITENCE

D – Dystrophia Myotonica

O – Osteogenesis Imperfecta

T
AUTOSOMAL DOMINANT INHERITENCE

D – Dystrophia Myotonica

O – Osteogenesis Imperfecta

M-

I – intermittent porphyria

T
AUTOSOMAL DOMINANT INHERITENCE

D – Dystrophia Myotonica

O – Osteogenesis Imperfecta

M-

I – intermittent porphyria

N – noonan syndrtome

T
AUTOSOMAL DOMINANT INHERITENCE

D – Dystrophia Myotonica

O – Osteogenesis Imperfecta

M-

I – intermittent porphyria

N – noonan syndrtome

A – achondroplasia

T
AUTOSOMAL DOMINANT INHERITENCE

D – Dystrophia Myotonica

O – Osteogenesis Imperfecta

M-

I – intermittent porphyria

N – noonan syndrtome

A – achondroplasia

N – neurofibromatosis

T
AUTOSOMAL DOMINANT INHERITENCE

D – Dystrophia Myotonica

O – Osteogenesis Imperfecta

M-

I – intermittent porphyria

N – noonan syndrtome

A – achondroplasia

N – neurofibromatosis

T – tuberous sclerosis / treacher collin syndrome


Autosomal recessive ?

H
Autosomal recessive ?

A: albinism / alkaptonuria

H
Autosomal recessive ?

A: albinism / alkaptonuria

B: beta thalassemia

H
Autosomal recessive ?

A: albinism / alkaptonuria

B: beta thalassemia

C: CAH, cystic fibrosis

H
Autosomal recessive ?

A: albinism / alkaptonuria

B: beta thalassemia

C: CAH, cystic fibrosis

D: deafness

H
Autosomal recessive ?

A: albinism / alkaptonuria

B: beta thalassemia

C: CAH, cystic fibrosis

D: deafness

E: emphysema

H
Autosomal recessive ?

A: albinism / alkaptonuria

B: beta thalassemia

C: CAH, cystic fibrosis

D: deafness

E: emphysema

F: friedrich’s ataxia

H
Autosomal recessive ?

A: albinism / alkaptonuria

B: beta thalassemia

C: CAH, cystic fibrosis

D: deafness

E: emphysema

F: friedrich’s ataxia

G: galactosemia, gaucher disease

H: homocysteinuria
Q: a 4 year old child with unexplained hepatosplenomegaly +

developmental delay with the following appearance in the

bone marrow ? What is your best diagnosis


Q: a 4 year old child with unexplained hepatosplenomegaly +

developmental delay with the following appearance in the bone

marrow ? What is your best diagnosis

A. Glucose 6 phosphatase

B. Glucocerebrosidase

C. Beta galactosidase

D. None of the above


Q: a 4 year old child with unexplained hepatosplenomegaly +

developmental delay with the following appearance in the bone

marrow ? What is your best diagnosis

A. Glucose 6 phosphatase

B. Glucocerebrosidase ( enzyme deficient ) gaucher cells in gaucher

disease

C. Beta galactosidase

D. None of the above


Q: a 4 year old child with unexplained hepatosplenomegaly +

developmental delay with the following appearance in the bone

marrow ? What is your best diagnosis

A. Glucose 6 phosphatase – von gierkes disease

B. Glucocerebrosidase ( enzyme deficient ) gaucher

C. Beta galactosidase – GM1 gangliosidosis

D. None of the above


A 6 year old child with pain abdomen with rashes over the lower limbs On

further examination patient had hematuria and proteinuria. What is your

best possible diagnosis ?

A. HUS

B. HSP

C. Nephrotic syndrome

D. SLE
A 6 year old child with pain abdomen with rashes over the lower limbs On

further examination patient had hematuria and proteinuria. What is your

best possible diagnosis ?

A. HUS

B. HSP

C. Nephrotic syndrome

D. SLE
Q21: A 6 year old child with abdominal pain &
red, elevated skin rashes, mainly involving the
lower limbs: what is your probable diagnosis:

NEET PG JAN 2019

A. Kawasaki

B. HSP

C. Varicella infection

D. meningococcemia

Ref: Nelson 20th p 1768,ghai 8/e p11 b


•HSP: Henoch
Schonlein
purpura
NEET
last 5 year questions
Dr.anand
You can’t start the next chapter of your life
You can’t start the next chapter of your life
If you are Re-reading the Last One !!
• Q: While giving a routine vaccination to a child, ward sister
has an open PENTAVALENT and open MR vaccine – which
one to discard?

A. Discard MR keep penta

B. Discard penta keep MR

C. Discard both

D. Keep both
• Open vial policy – how long we can use a vaccine ?
• Open vial policy – how long we can use a vaccine

• Expiry date has not reached


• Open vial policy – how long we can use a vaccine

• Expiry date has not reached

• Cold chain has been maintained


• Open vial policy – how long we can use a vaccine

• Expiry date has not reached

• Cold chain has been maintained

• Date of manufacturing is written clearly


• Open vial policy – how long we can use a vaccine

• Expiry date has not reached

• Cold chain has been maintained

• Date of manufacturing is written clearly

• Open vial policy is applicable for:


• Open vial policy is applicable for:

• TT
• Open vial policy is applicable for:

• TT

• DPT / PENTAVLENT VACCINE


• Open vial policy is applicable for:

• TT

• DPT

• PCV
• Open vial policy is applicable for:

• TT

• DPT

• PCV

• Hep B

• POLIO
• Open vial policy is NOT applicable for:
• Open vial policy is NOT applicable for:

• JAPENESE ENCEPHALITIS
• Open vial policy is NOT applicable for:

• JAPENESE ENCEPHALITIS

• BCG
• Open vial policy is NOT applicable for:

• JAPENESE ENCEPHALITIS

• BCG

• ROTA
• Open vial policy is NOT applicable for:

• JAPENESE ENCEPHALITIS

• BCG

• ROTA

• Measles / MR
• Q: While giving a routine vaccination to a child, ward sister
has an open PENTAVALENT and open MR vaccine – what is
the best way to discard them ?

A. Discard MR keep penta

B. Discard penta keep MR

C. Discard both

D. Keep both
• Q: during a one minute APGAR score – the baby had 110 beats
per minute, lusty cry(STRONG ENERGETIC), acrocyanosis,
minimal flexion to stimulation. What is the best APGAR score ?

A. 5

B. 6

C. 7

D. 8

E. 9
• Q: during a one minute APGAR score – the baby had 110 beats
per minute, lusty cry(STRONG ENERGETIC), acrocyanosis,
minimal flexion to stimulation. What is the best APGAR score ?

A. 5

B. 6

C. 7

D. 8

E. 9
• Q: 5 year old child with cough and cold was given antibiotic for 7 days. On
further follow up patient had elevated ESR with polyarthritis with no
features of carditis. In the prophylaxis treatment of this disease – what is
the best option available ?

A. Injection benzathine penicillin every 21 days IM for 5 years or until they


are 18 years of age which ever is later

B. Injection benzathine penicillin every 21 days IM for 10 years or until they


are 25 years of age which ever is later

C. both options are possible

D. None of the above


• Q: 5 year old child with cough and cold was given antibiotic for 7 days. On
further follow up patient had elevated ESR with polyarthritis with no
features of carditis. In the prophylaxis treatment of this disease – what is
the best option available ?

A. Injection benzathine penicillin every 21 days IM for 5 years or until they


are 18 years of age which ever is later

B. Injection benzathine penicillin every 21 days IM for 10 years or until they


are 25 years of age which ever is later

C. both options are possible

D. None of the above


• Till how long to give the injection ?

• Patient without proven carditis


should receive prophylaxis for 5 years
after the last episode, or until they
are 18-yr-old (whichever is longer)
• Till how long to give the injection ?

• Patient WITH proven carditis should


receive prophylaxis for 10 YEARS after
the last episode, or until they are 25-
yr-old (whichever is longer)
Q: a 12 months old child came with complaints of pain
abdomen, palpable mass & x ray showing following
appearance ?

A. CHPS

B. Duodenal atresia

C. Jejunal atresia

D. Intusussception
Q: a 12 months old child came with complaints of pain
abdomen, palpable mass & x ray showing following
appearance ?

A. CHPS

B. Duodenal atresia

C. Jejunal atresia

D. Intusussception
•Genetics
Identify the

syndrome?

A. Down syndrome

B. Patau syndrome

C. Edward syndrome

D. Turner syndrome
Edward Syndrome
Trisomy 18
• R: rocker bottom feet

• O: overlapping fingers

• C: cardiac defects

• K: kidney defects

• Y: microcephaly

• Mountain: mental retardation


What is rocker bottom ?
Convexity of soles like a rocking chair
Chromosome involved ?

A. Down syndrome

B. Patau syndrome

C. Edward syndrome
Chromosome involved ?
A. Down syndrome Chromosome no.21

B. Patau syndrome 13

C. Edward syndrome 18
Identify the syndrome ?
Mongoloid slant

Epicanthal folds

Single palmar

crease

Sandle gap

DOWN SYNDROME
How many chromosomes are there in a cell ?
Human cells have 23 pairs of chromosomes (22 pairs
of autosomes + one pair of sex chromosomes), giving
a total of 46 per cell.
Cause:

• Most common ?
Cause:

• Non disjunction:95% - means


chromosome don’t split apart
•E

•E

•E
• Eyes – Brushfield spots in iris

• Epicanthal folds

• Eyes mongoloid slant


•C

•C

•C
• Cong. Heart – most specific endocardial cushion

defect, most common ostium primum ASD, VSD

• Congenital hypothyroidism

• Clinodactyly
•S
•S
•S
• Simian crease
• Sandal gap
• Short head brachycephaly
Clinodactyly
Curvature of
the digit
Brushfield spots
Moro reflex is absent in ?

GIT: most common ?


Moro reflex is absent in ?

GIT: most common ?

Males are always ?


Moro reflex is absent in ?

GIT: most common ?

Males are always ?

In down IQ normal or not ?


Moro’s reflex is absent in down syndrome

GIT: duodenal atresia, H disease.

Males are always infertile

In down IQ is < 50

Normal is > 70
Moro’s reflex is absent in down syndrome

GIT: duodenal atresia, H disease.

Males are always infertile

In down IQ is < 50

Normal is > 70
Which is the syndrome where IQ will

not be affected ?
• Which is the syndrome where IQ will not be
affected ?
• TURNER SYNDROME
• Most common heart disease in

down: ?

• Most common type of overall ASD: ?


• Most common heart disease in

down: Endocardial cushion defect /

ASD( 40-60% )

• Most common type of overall ASD:

Ostium secundum
How do you screen ?
Method of ante natal screening
for down syndrome
• 1st trimester:
Screening method

Maternal age + biochemical

markers (B-HcG, PAPPA)

Maternal age + radiology ( USG-

NT >3mm)
Ref: Nelson 20th p a

Q76: in the first trimester screening for down syndrome which of

the following test is to be done:

NOV 2016

A. Beta HCG, PAPPA

B. MSAFP, PAPPA

C. Inhibin, PAPPA

D. Unconj. Estriol, PAPPA


Ref: Nelson 20th p a

Q76: in the first trimester screening for down syndrome which of

the following test is to be done:

NOV 2016

A. Beta HCG, PAPPA

B. MSAFP, PAPPA

C. Inhibin, PAPPA

D. Unconj. Estriol, PAPPA


• Pregnancy associated with plasma protein A
• 1st trimester - nuchal thickness ?
• 1st trimester - Nuchal thickness > 3 mm
• Blood sample of mother
• 2nd trimester
• 2nd trimester
Screening method

Triple screen*

Quadruple scan*
• 2nd trimester

Screening method

Triple screen* (AFP, Unconjugated

estriol, B HCG

Quadruple scan*

+ INHIBIN A
AFP Estriol HCG Inhibin A

Down Low Low High High High

Turner Low Low Very high Very high High

Edward Unchang Low Very low Unchang HE is low


ed ed

Patau Increased Normal Normal Normal AFPatau


is high
•Special feature
Broad thumb + great toe mediated medially
Rubinstein Taybi Syndrome
Absent clavicle
Clediocranial
dysostosis
Heterochromic Iridis
Heterochromia iridis: Iris in one eye has a different
color than the iris of the other eye.
Colured part of the eye
• Waardenburg syndrome

• Hearing loss

• Distinctive hair coloring

• Heterochromic iridis
Turner syndrome Noonan Syndrome

Karyotype 45 X0 Karyotype Normal

Infertile due to streak Fertile but delayed


ovaries puberty

Only Females Males or females

Intelligence Normal Intellectual Disability


Present

Heart: Pulmon.Stenosis

Anti mongoloid slant


Turner: she is turning a wheel,
driver is short
& prefer LOW CABINS
Turner: she is turning a wheel,
driver is short
& prefer LOW CABINS
Tuner Syndrome: LOW CABINS

L: low thyroid, lymphedema

O: Ovaries under developed,

streak ovaries

W: webbed neck
C: cystic hygroma

A: primary amenorrhea

Bi: bicuspid aortic valve

( MC heart disease in Turner)

absent Barr body,

N: nipples widely spaced

S: short stature, sensorineural hearing loss,

short 4th metacarpal


• Most common cause of death in turner

is COA
Which condition you see MADE LUNG’S
deformity ?
Which condition you see MADE LUNG’S
deformity ?

Abnormal growth of the wrist when child


is born
• Mental retardation ?

• Lymhedema ?

• Short stature ?
Mental retardation -

Lymhedema +

Short stature +
AFP Estriol HCG Inhibin A

Down Low Low High High High

Turner Low Low Very high Very high High

Edward Unchang Low Very low Unchang HE is low


ed ed

Patau Increased Normal Normal Normal AFPatau


is high
You see high FSH and LH

Due to streak ovaries +ve feedback is sent to


pituitary gland to secrete more and more FSH LH
Clinodactyly: Curvature of the digit mostly the 5th finger
• Treatment of choice

• Pediatric Radiology

• Triad

• One liners

• Drug dosage

• Mnemonics
TREATMENT OF CHOICE-
1 liners
• Absence seizures: Ethosuximide, 2nd choice: Sodium Valproate
• Acute diarrhea
Acute diarrhea: WHO ORS ZINC
Do you give antibiotics in diarrhea?
Diarrhea with SAM
Diarrhea with Dysentery
Diarrhea with rice watery stool
What is tenesmus ?
What is dysentery ?
Frothy diarrhea with perianal excoriation ?
Types of E.Coli causing diarrhea ?
Whenever I accompany him to the toilet, he
passes large bulky foul smelling stool which
fixes to the pan!!
Entero toxigenic
Entero invasive – blood in stool
Entero hemorrhagic
Entero pathogenic – malabsorption
diarrhea
Entero secretory
ACUTE EPIGLOTTITIS:
IV Ceftriaxone (3rd gen. cephalosporins)
Cholera:
Cholera: doxycycline/ azithromycin
• Croup (laryngotracheobronchitis)
Croup (laryngotracheobronchitis):
Dexamethasone + - nebulized epinephrine
• Foreign Body Esophagus
Foreign Body Esophagus: Endoscopic visualization
and removal
• Absolute contraindication of breast feeding ?
Galactosemia: lactose free diet
Breast feeding contraindicated
Nocturnal Enuresis:
• Definition

• Types

• Treatment
• Definition: Involuntary voiding at night after 5 yrs of age
• Types: Primary (MC) never been dry at night, secondary (6
months )
• 50% boys, family history +ve
• If 1 parent has +ve, then each child has 44% risk of NE
• If both parents are +ve: each child has 77% risk of NE
• Rx: Reissuance
• Fluid intake restricted after 6 pm
• Caffeine, sugar avoid after 4 pm
• Void bedtime
§ Simplest Initial Measure: MOTIVATIONAL THERAPY (‘ STAR
CHART’)

§ Alarm therapy: success 30-60%

§ Drugs:

§ Rx of choice is conditioning therapy: Alarm therapy

§ Drug of choice 2nd line but not curative: Desmopressin(ADH) oral


bed time (effective in 40%)

§ Resistant/ overactive bladder- oxybutynin anticholinergic


antispasmodic

§ 3rd line: imipramine (tricyclic antidepressant)


Nocturnal enuresis:
Motivational alarm therapy
Nutritional rickets:
Nutritional rickets: Vit D3
Pertussis:
Pertussis: Azithromycin
HYALINE MEMBRANE DISEASE
RDS (HMD): Surfactant, CPAP
Meconium aspiration

syndrome.

The lungs are

overinflated with coarse

opacities throughout the

lungs.
Scurvy
•Scurvy: Vitamin C
•Wilson disease:
Wilson disease:

Cu chelating agents

Zinc, ammonium trithyomolybdate (trientine)


• Remain calm & focused
•\
Sab theek hojayega !
Failure Is The Key Path To Success
Be Humble – We All have A Past !
Just because you fail once, doesn't mean you gonna fail at
everything
CHAMPAGNE GLASS PELVIS
CHAMPAGNE GLASS PELVIS: Achondroplasia
• Claw sign or coiled spring sign
Caw sign or coiled spring sign: Intussusception
Cupping

Fraying

Splaying

RICKETS
FOOTBALL SIGN
FOOTBALL SIGN: pneumoperitoneum stage 3 NEC
Hair on end: thalassemia
Puff of smoke appearance
Puff of smoke appearance: Moyamoya disease
Bitot's spots are the build-up of keratin located
superficially in the conjunctiva of human´s eyes.

VITAMIN A DEFICIENCY
X1A

X1B

X2

X3A

X3B
X1A Conjunctival
xerosis
X1B

X2

X3A

X3B
X1A Conjunctival
xerosis
X1B Bitot spots

X2

X3A

X3B
X1A Conjunctival
xerosis
X1B Bitot spots

X2 corneal xerosis

X3A

X3B
X1A Conjunctival
xerosis
X1B Bitot spots

X2 Corneal xerosis

X3A Corneal ulceration


in <1/3rd of corna

X3B Corneal ulceration


in >1/3rd of corna
XN

XS

XF
XN NIGHT BLINDNESS

XS SCARRING

XF FUNDAL CHANGES
Blue sclera with h/o recurrent #
a)Bitot spots
b)Alkaptonuria
c)OI
D)Ehler Danlos syndrome
Ehler Danlos syndrome
Spot diagnosis!
Rachitic rosary / scorbutic rosary
Vitamin D / vitamin C
Painless / painful
Beaded, rounded / pointed
Do you like her necklace ?
Niacin(vit b3)
deficiency
Pellagra
What are the 4 Ds?

Do you like her necklace ?


• Diarrhoea

• Dermatitis: A scaly rash on skin exposed to light or trauma;

• Dementia: Mental disorientation, delusions and depression; and

• Death.
Name drugs which can be given
INTRATRACHEAL ?
Name drugs which can be given INTRATRACHEAL
? - NASAL

• NALOXONE
• ADRENALINE
• SURFACTANT
• ATROPINE

• LIGNOCAINE
What is the normal blood pressure of a 4 year
old?
•Systolic: 90 + age * 2
•Diastolic: 55 + age * 2
•Ans: 98/63 mm Hg
What is MY Age?
SITTING WITH TRUE TRIPOD SUPPORT 6 MONTHS
Which is the grade of hypothermia?
Mild Hypothermia
Soto’s syndrome
Eye of tiger sign
• Eye of tiger sign: PKAN
• Penthothenate kinase associated neurodegeneration /
hallervorden spatz syndrome
• Tram track sign: Sturge weber syndrome
Gastroschisis
Geographic tongue: folate or vit b 12 def.
Shamroth sign
• CTEV
•World Autism Day ?
87
88
90
Microcephaly
Squint

Low set ears


• What is arthritis
• Mono
• Oligo
• Poly
• What is arthritis
• Mono 1
• Oligo <4
• Poly >4
• Large joint
• Small joint
• Fleeting type
• Night cry and bony tenderness
• Large joint - rheumatic
• Small joint - rheumatoid
• Fleeting type - rheumatic
• Night cry and bony tenderness – leukemia
• Trident hand is seen in ACHONDROPLASIA
• Phocomelia
• It is because of which drug intake by mother ?
•THALIDOMIDE
• Identify the sign ?
Trousseau sign: The hand adopts a characteristic

posture (spasm) when the sphygmomanometer cuff is

inflated above the systolic blood pressure within 5

minutes

It is a test for neuromuscular excitability which helps in

detecting latent tetany


• IDENTIFY THE SYNDROME ?
• Child has mental retardation
• FRAGILE X SYNDROME
• Mental retardation
• Autosomal recessive
• Delayed milestones
• Mandibular prognathism
• Seizures
• Triple nucleotide
repeat mutation defect
•1st most common cause of mental retardation
is _______ ?

•2nd – ______ ?
• 1st most common cause of mental retardation is
DOWN SYNDROME

• 2nd – Fragile X syndrome

These are not preventable causes


Preventable cause of mental retardation is congenital
hypothyroidism
•PG 2019 QUESTIONS
1) Bidextrous approach comes at what age ?

NEET PG 2019
1) Bidextrous approach comes at what age ?
4 MONTHS
NEET PG 2019
BMU
456
Q2: What is the interpretation of APGAR SCORE 3:

NEET PG 2019

A. Moderately depressed baby

B. Severely depressed baby

C. Normal

D. Mild birth asphyxia


Score 0-3

4-7

>7
Score 0-3 Severe birth asphyxia

4-7 Moderate birth asphyxia

>7 Normal
Q2: What is the interpretation of APGAR SCORE 3:

NEET PG 2019

A. Moderately depressed baby

B. Severely depressed baby

C. Normal

D. Mild birth asphyxia


Q3: Where to look for pre ductal oxygen saturation in PDA

in a 3 minute born infant ?

NEET 2019 Question


1. Left upper limb

2. Left lower limb

3. Right upper limb

4. Right lower limb


Q3: Where to look for pre ductal oxygen saturation in PDA

in a 3 minute born infant ?

NEET 2019 Question


1. Left upper limb

2. Left lower limb

3. Right upper limb

4. Right lower limb


Q4: Which of the following is seen in CYSTIC FIBROSIS:

NEET PG JANUARY 2019

A. Low sweat chloride level

B. Elevated sweat chloride level

C. Low sweat sodium

D. Elevated sweat potassium level


Sweat ducts normally absorb sodium from lumen which is decrease in
CF causing salty sweat
( high sodium and chloride) HALLMARK OF CF
Sweat chloride level is >60mEq/L in cystic fibrosis
Q4: Which of the following is seen in CYSTIC FIBROSIS:

NEET PG JANUARY 2019

A. Low sweat chloride level

B. Elevated sweat chloride

C. Low sweat sodium

D. Elevated sweat potassium level


• Sweat ducts are not obstructed

• Other ducts are obstructed due to stasis of secretions


Q5: Which of the following exocrine glandular ducts is

not obstructed in cystic fibrosis ?

NEET 2019 Question

1. Pancreas

2. Lung

3. Sweat glands

4. All if the above


Q5: Which of the following exocrine glandular ducts is

not obstructed in cystic fibrosis ?

NEET 2019 Question

1. Pancreas

2. Lung

3. Sweat glands

4. All if the above


Q6: Which of the fontanelle is last to close:

NEET PATTERN JAN 2018

A. Posterior fontanelle

B. Anterior fontanelle

C. Mastoid fontanelle

D. Sphenoidal

Fontanelle
• Anterior fontanelle: 10-24mnths
• Posterior: 2-3 months

• Sphenoidal: 6 months
• Mastoid: 6-18 months
Q6: Which of the fontanelle is last to close:

NEET PATTERN JAN 2018

A. Posterior fontanelle

B. Anterior fontanelle

C. Mastoid fontanelle

D. Sphenoidal

Fontanelle
Q7. In which of the diseases is the following appearance of

calves seen in a child?

AIIMS NOV 2016

a. Spinal muscular atrophy

b. Myopathy

c. Muscular dystrophy

d. Peripheral neuropathy
Q7. In which of the diseases is the following appearance of

calves seen in a child?

AIIMS NOV 2016

a. Spinal muscular atrophy

b. Myopathy

c. Muscular dystrophy

d. Peripheral neuropathy
Q:8
FMGE PATTERN
• Gene mutation in DMD ?
• Gene mutation in DMD ?

• Xp21 dystrophin gene


Is it proximal or distal myopathy ?
Is it proximal or distal myopathy ?

Ans: proximal
• Most common cause of death in
DMD?
• Most common cause of death in
DMD?

• Respiratory failure during sleep


Q9: Which of the following medicines taken during pregnancy is

responsible for the defect seen in the child below?

AIIMS 2016

a. Ferrous sulfate

b. Isotretinoin

c. Folic acid

d. Metoclopramide
Q9: Which of the following medicines taken during pregnancy is

responsible for the defect seen in the child below?

AIIMS 2016

a. Ferrous sulfate

b. Isotretinoin

c. Folic acid

d. Metoclopramide
10)Anti malarial:
Anti malarial:
11)Cocaine:
11)Cocaine:
12)Lithium
12)Lithium
TOF
• TRIOLOGY

• TETROLOGY

• PENTOLOGY
•TRIOLOGY - RAP

•TETROLOGY

•PENTOLOGY
In Fetal circulation
Remnants of fetal circulation

Structure Remnant

Ductus arteriosus

Umbilical artery

Umbilical vein

Ductus venosus
Remnants of fetal circulation

Structure Remnant

Ductus arteriosus Ligamentum arteriosum

Umbilical artery Medial umbilical


ligament

Umbilical vein Ligamentum teres


hepatis

Ductus venosus Ligamentum venosum


•A
•B
In Fetal circulation

4 chambered structure of heart develops by


____weeks of IU life
In Fetal circulation

4 chambered structure of heart develops by 6


weeks of IU life
Changes in the circulation after birth
vDuctus venosus closes by 7 days of life
vClosure of DA:
vFunctional closure: immediately 10 – 15 hrs after
birth due to contraction of smooth muscles
vAnatomic closure: 10-21 days of life due to intimal
thickening
Foramen ovale – months to years
• I want to keep the PDA open ?
• I want to close the PDA ?
• I want to keep the PDA open ?
• PGE1 analogue / Alprostadil
• I want to close the PDA ?
• Indomethacin / Iburofen
Indomethacin and ibuprofen are not effective in full term
children.
Q13: Palivizumab is a humanized monoclonal antibody. For

which of the following conditions has it been approved for?

AIIMS NOV 2016

a. Avian influenzae

b. Parainfluenzae

c. Respiratory syncytial virus

d. Coxsackie virus
vBronchiolitis:

v Cause: RSV(most common)

v Age gp: 1-6 months but can affect up to 2-3 yrs*

v X-ray: hyperinflation & infiltrates

v Main stay of rx: MOIST o2 Inhalation

v Beta adrenergic & ipratropium are NOT recommended <6 months

v Ribavirin- antiviral ( it shortens the course of illness in HIV pt)

v RSV Ig & palivizumab: should be considered for infants with chronic lung

disease, prematurity.
Q13: Palivizumab is a humanized monoclonal antibody. For

which of the following conditions has it been approved for?

AIIMS NOV 2016

a. Avian influenzae

b. Parainfluenzae

c. Respiratory syncytial virus

d. Coxsackie virus
14) IDENTIFY THE INSTRUMENT?
HARPENDEN CALLIPER
15. A 6 days old neonate weighing 2800 gm (birth weight 3000 gm)
was brought with the complaints of fever, poor feeding and poor
activity. There was no history of vomiting or diarrhea. Axillary
temperature was 39°C with depressed fontenalle, sunken eyes,
decreased urine output and decreased skin turgor. Her mother has
the history of decreased milk production. What is your diagnosis?
AIIMS NOV 2016

a. Neonatal sepsis

b. Galactosemia

c. Fever & dehydration

d. Acute renal failure


INVESTIGATIONS
➤ BLOOD CULTURE: GOLD STANDARD (Confirmatory)
➤ SEPTIC SCREEN: ANY 2 OR MORE THAN 2

TOTAL LEUCOCYTE
TOTAL COUNT
LEUCOCYTE COUNT <5000/cumm
<5000/cumm

ABSOLUTE
ABSOLUTE NEUTROPHIL
Low
Low -- AS
ASPER
PER MANROE
MANROE CHART
FOR TERM AND MOUZINHO’S
NEUTROPHIL COUNT
COUNT CHART FOR TERM
CHART FOR VLBW
AND MOUZINHO’S
IMMATURE/TOTAL
IMMATURE/TOTAL
>0.2
>0.2
NEUTROPHIL
NEUTROPHIL

>15MM IN FIRST
MICRO
MICRO ESR
ESR >15MM IN FIRST HOUR
HOUR

CRP
CRP >I >Img/dl
mg/dl(+ve)
(+ve)
15. A 6 days old neonate weighing 2800 gm (birth weight 3000 gm)
was brought with the complaints of fever, poor feeding and poor
activity. There was no history of vomiting or diarrhea. Axillary
temperature was 39°C with depressed fontenalle, sunken eyes,
decreased urine output and decreased skin turgor. Her mother has
the history of decreased milk production. What is your diagnosis?

AIIMS NOV 2016

a. Neonatal sepsis

b. Galactosemia

c. Fever & dehydration

d. Acute renal failure


Q16. A neonate presented with cicatrizing skin lesions all over the body with

hypoplasia of all limbs. An MRI of the brain revealed diffuse cerebral atrophy. An

ophthalmologic evaluation reveals chorioretinitis. Which of these tests is most

likely to show a positive result in this patient?

AIIMS NOV 2016

a. Anti-HCMV antibodies

b. Anti-toxoplasma antibodies

c. Anti-VZV antibody

d. Anti-rubella antibody
• Limb hypoplasia

• Cicatrical skin lesions

• h/o fever with rash in end

of 1st trimester

• Chorioretinitis

Congenital varicella syndrome


Q16. A neonate presented with cicatrizing skin lesions all over the body with

hypoplasia of all limbs. An MRI of the brain revealed diffuse cerebral atrophy. An

ophthalmologic evaluation reveals chorioretinitis. Which of these tests is most

likely to show a positive result in this patient?

AIIMS NOV 2016

a. Anti-HCMV antibodies

b. Anti-toxoplasma antibodies

c. Anti-VZV antibody

d. Anti-rubella antibody
Q17 . What should be the ideal temperature in delivery room for

the neonates to be kept in warmer? AIIMS NOV 2016

a 22-26°C

b 28-30°C

c. 30-35°C

d. 37°C
Q17 . What should be the ideal temperature in delivery room for

the neonates to be kept in warmer? AIIMS NOV 2016

a 22-26°C

b 28-30°C

c. 30-35°C

d. 37°C
Q18. Ponderal index is:

AIIMS NOV 2016

a. Square root of height in feet by weight in grams

b. Weight in kilograms by cube of height in meters

c. Mid-upper arm circumference to head circumference

ratio

d. Head circumference to abdominal circumference ratio


Q18. Ponderal index is:

AIIMS NOV 2016

a. Square root of height in feet by weight in grams


b. Weight in kilograms by cube of height in meters
c. Mid-upper arm circumference to head circumference
ratio
d. Head circumference to abdominal circumference ratio
Q19. Ideal route of drug delivery in neonatal resuscitation

is:

AIIMS NOV 2016

a. Intraosseous

b. Through umbilical vein

c. Through peripheral vein

d. Through umbilical artery


Q19. Ideal route of drug delivery in neonatal resuscitation

is:

AIIMS NOV 2016

a. Intraosseous

b. Through umbilical vein

c. Through peripheral vein

d. Through umbilical artery


• WHICH ARE THE DRUGS GIVEN INTRATRACHEAL ?
• WHICH ARE THE DRUGS GIVEN INTRATRACHEAL ?

•N

•A

•S

•A

•L
• WHICH ARE THE DRUGS GIVEN INTRATRACHEAL ?

• N - NALOXONE

• A - ATROPINE

• S – SURFACTANT

• A – ADRENALINE

• L - LIGNOCAINE
Q20. A neonate presented with jaundice on first day of life. His

mother’s blood group is ‘O’ positive. How will you manage this

patient?

AIIMS NOV 2016

a. Observe only as it is mostly physiological jaundice

b. Exchange transfusion

c. Liver function tests and liver biopsy as it is mostly due to

cholestasis

d. Phototherapy
Q20: A neonate presented with jaundice on first day of life. His

mother’s blood group is ‘O’ positive. How will you manage this

patient? AIIMS NOV 2016

a. Observe only as it is mostly physiological jaundice

b. Exchange transfusion

c. Liver function tests and liver biopsy as it is mostly due to

cholestasis

d. Phototherapy
• WHAT ARE THE COMPLICATIONS OF
PHOTOTHERAPY ?
• Insensible water loss

• Diarrhea

• Inc. incidence of PDA

• Retinal damage

• Hypoglycemia
Any syndrome it causes ?
Bronze baby syndrome
Dark, grayish brown discoloration
Q21: What is the recommended dose of steroids for

attaining fetal lung maturity? AIIMS NOV 2016

a. Inj. betamethasone 12 mg for 2 doses 12 hours apart

b. Inj. betamethasone 12 mg for 2 doses 24 hours apart

c. Inj. dexamethasone 6 mg for 4 doses 24 hours apart

d. Inj. dexamethasone 12 mg for 2 doses 12 hours apart


• Inj Betamethasone 12 mg IM 2 doses, 24

hours apart

• Inj Dexamethasone 6 mg IM every 12 hrs for 4

doses

• 12 * 2 = 24
Do you give steroids to the mother or the baby ?

Route ?
Q21: What is the recommended dose of steroids for

attaining fetal lung maturity? AIIMS NOV 2016

a. Inj. betamethasone 12 mg for 2 doses 12 hours apart

b. Inj. betamethasone 12 mg for 2 doses 24 hours apart

c. Inj. dexamethasone 6 mg for 4 doses 24 hours apart

d. Inj. dexamethasone 12 mg for 2 doses 12 hours apart


Q22: Identify the syndrome
Q23: All of the following are sequelae of fetal alcohol syndrome

except:

AIIMS MAY 2017

a. Macrocephaly

b. Microcephaly

c. Holoprosencephaly

d. Short stature
Q23: All of the following are sequelae of fetal alcohol syndrome

except:

AIIMS MAY 2017

a. Macrocephaly

b. Microcephaly

c. Holoprosencephaly

d. Short stature
24)
Identify the Anomaly ?
24)
Identify the abnormality ?

Holoprosencephaly
Q25: which of the following is abnormal :

NEET PG JAN 2019

A. RR>40 pm in child 2-12 months of age

B. RR > 50 cpm in a child between 2-12 months of


age

C. RR > 30cpm in a child between 1-5 yrs of age

D. RR > 50cpm in a child <2 months of age

Ref: Nelson 20/e p 2036, ghai 8/e p 369b


WHO CUT OFF OF FAST BREATHING

• 0 – 2 months >60

• 3 – 1 year >50

• 1-5 yr >40

• >5 yr > 30
Q25: which of the following is abnormal :

NEET PG JAN 2019

A. RR>40 pm in child 2-12 months of age

B. RR > 50 cpm in a child between 3-12


months of age

C. RR > 30cpm in a child between 1-5 yrs of


age

D. RR > 50cpm in a child <2 months of age

Ref: Nelson 20/e p 2036, ghai 8/e p 369b


Q26) WHAT IS THE NORMAL HC OF A 2 MONTH OLD IF THE
BIRTH HC IS 33 CM ? NEET 2018

a) 34 cm

b) 35 cm

c) 36 cm

d) 37 cm
• What is the normal head circumference?

• 33-35cm

0-3 months 2cm/month

3-6 months 1 cm/ month

7-12 months 0.5 cm/ month

1-3 yr 0.2/month
Q26) WHAT IS THE NORMAL HC OF A 2 MONTH OLD IF THE
BIRTH HC IS 33 CM ? NEET 2018

a) 34 cm

b) 35 cm

c) 36 cm

d) 37 cm
27) IDENTIFY THE INSTRUMENT
27) IDENTIFY THE INSTRUMENT: INFANTOMETER
28) SPOT THE FINDING & RX
28) SPOT THE FINDING & RX
HYDROCEPHALUS , VP SHUNT
29) SAM IS DEFINED AS ?
Q29: Definition of SAM include which of the following:

NEET PG JAN 2019

A. Weight for height <-2sd

B. Weight for height <-3sd

C. Weight for age <-2sd

D. Weight of age <-3sd


Q29: Definition of SAM include which of the following:

NEET PG JAN 2019

A. Weight for height <-2sd

B. Weight for height <-3sd

C. Weight for age <-2sd

D. Weight of age <-3sd


30) IDENTIFY THE SYNDROME

e
30) EDWARD SYNDROME

ROCKY M

e
31. Dried blood spot test in neonates is used for:

AIIMS NOV 2017

a. Inborn error of metabolism

b. Blood group

c. Total cell count

d. Creatinine and bilirubin


31. Dried blood spot test in neonates is used for:

AIIMS NOV 2017

a. Inborn error of metabolism

b. Blood group

c. Total cell count

d. Creatinine and bilirubin


32. A 4-days-old neonate is brought by the mother to
the pediatrician and is found to have a bilirubin of 18
g/dl. What is the next step?

AIIMS NOV 2017

a. Stop breastfeeding

b. Start phototherapy and continue breastfeeding

c. Start phototherapy and stop breastfeeding

d. Exchange transfusion
32. A 4-days-old neonate is brought by the mother to
the pediatrician and is found to have a bilirubin of 18
g/dl. What is the next step?

AIIMS NOV 2017

a. Stop breastfeeding

b. Start phototherapy and continue breastfeeding

c. Start phototherapy and stop breastfeeding

d. Exchange transfusion
• Indication of exchange transfusion are:

• Cord hemoglobin <10 g/dl or cord bilirubin >5

mg/dl

• DCT +ve

• H/O kernicterus or Erythroblastosis in sibling


33. Hutchinson’s triad in congenital syphilis include:

AIIMS Nov 2017

a. Interstitial keratitis, eighth nerve involvement, Hutchinson’s

teeth

b. Interstitial keratitis, mulberry teeth, periostitis

c. Periostitis, mulberry teeth, eighth nerve involvement

d. Hutchinson’s teeth, eighth nerve involvement, periostitis


33. Hutchinson’s triad in congenital syphilis include:

AIIMS Nov 2017

a. Interstitial keratitis, eighth nerve involvement, Hutchinson’s

teeth

b. Interstitial keratitis, mulberry teeth, periostitis

c. Periostitis, mulberry teeth, eighth nerve involvement

d. Hutchinson’s teeth, eighth nerve involvement, periostitis


34. Patient with growth retardation, delayed

milestones with kinky hair. What is the diagnosis?

AIIMS NOV 2017

a. Menke’s disease b. Trisomy 21

c. Lesch nyhan syndrome d. Wilson disease


34. Patient with growth retardation, delayed

milestones with kinky hair. What is the diagnosis?

AIIMS NOV 2017

a. Menke’s disease b. Trisomy 21

c. Lesch nyhan syndrome d. Wilson disease


Trichorrhexis nodosa: Defect in the
hair shaft
weak points (nodes)
that cause the hair to
break off easily.
• Pili torti is a rare hair condition characterized by
fragile hair.
Cutis Laxa: In which the skin becomes inelastic and hangs
loosely in folds. Patients develop a prematurely aged
appearance.
• ATP7A

• ATP7B
• ATP7A - Menke

• ATP7B – Wilson
Q35: The mile stone of a 3 year old child are
considered delayed if he is unable to:

AIIMS NOV 2017

A. Hop on 1 foot

B. Use spoon effectively

C. Copy a square

D. Reliably catch a boll

Ref: Nelson 9/e 20/e p 75b


A. Hop on 1 foot: 4 yr.

B. Child is able to to feed self at 18 months & Use

spoon effectively: 24 months(2yr)

C. Copy a square: 4 yr.

D. Reliably catch a boll: 4 yr.


36.What is the sequence in which following mile stones
are attained?

A. Abcd

B. Dcba

C. Dcab

D. Dbca

AIIMS PATTERN 2018


12

A. Abcd

B. Dcba

C. Dcab
15
A

D. Dbca

6
• D feet to mouth 5 months

• C Sits with support 6 months

• A walk with support 12 months

• B creeps upstairs 15 months


37: At what age child
can do this ?
A. 3yr
B. 4yr
C. 5yr
D. 6yr
37: At what age child
can do this ?
A. 3yr
B. 4yr
C. 5yr
D. 6yr
38.When can I
child walk
backwards?
AIIMS PG 2016 39.Tricycle,
hopping &
A. 2 yrs. skipping?
B. 3yr AIIMS JUNE 2017
C. 4yr A. 3,4,5
D. 5yr B. 4,5,6
C. 4,3,5
D. 5,4,3
38.When can I
child walk
backwards?
AIIMS PG 2016 39.Tricycle,
hopping &
A. 2 yrs. skipping?
B. 3yr AIIMS JUNE 2017
C. 4yr A. 3,4,5
D. 5yr B. 4,5,6
C. 4,3,5
D. 5,4,3
40) Tower of 6 cubes is made at what age ? AIIMS 2016
41) Which game the child is playing and when
Does it come ? AIIMS 2017
42) AIIMS 2017
Which no. Blue colour IV line ?
42) AIIMS 2017
Which no. Blue colour IV line ?
43) A coin is loaded in the esophagus of

An asymptomatic 3 yr old child, Rx is ?

AIIMS 2016

1. Endoscopic removal within 24 hrs

2. Immediate endoscopic removal

3. Wait for 48 hrs

4. Dislodge the coin by using ryle’s tube


• Asymptomatic objects like coin can be observed upto 24 hrs

• If they still remain lodges in esophagus, endoscopic removal is

warranted.
43) A coin is loaded in the esophagus of

An asymptomatic 3 yr old child, Rx is ?

AIIMS 2016

1. Endoscopic removal within 24 hrs

2. Immediate endoscopic removal

3. Wait for 48 hrs

4. Dislodge the coin by using ryle’s tube


44. A child to the emergency with a history of
ingestion of button battery, on X-ray it was found in
the stomach or duodenum. What is the next step?

AIIMS MAY 2018

a. Endoscopic removal of battery

b. Wait and watch

c. Repeat X-ray after 5 days

d. Immediate laparotomy
44. A child to the emergency with a history of
ingestion of button battery, on X-ray it was found in
the stomach or duodenum. What is the next step?

AIIMS MAY 2018

a. Endoscopic removal of battery

b. Wait and watch

c. Repeat X-ray after 5 days

d. Immediate laparotomy
45. APGAR stands for

AIIMS NOV 2017

a. Appearance, Pressure, Grimace, Activity respiration

b. Appearance, Pulse, Grimace, Activity, Respiration

c. Appearance, Pressure, Grimace, Activity, Respiration

d. Awareness, Pulse, Grimace, Activity, Respiration


45. APGAR stands for

AIIMS NOV 2017

a. Appearance, Pressure, Grimace, Activity respiration

b. Appearance, Pulse, Grimace, Activity, Respiration

c. Appearance, Pressure, Grimace, Activity, Respiration

d. Awareness, Pulse, Grimace, Activity, Respiration


46. After the delivery of an infant of diabetic mother, blood glucose

of the infant was 60 mg/dl. Which other investigation does the

sister expects that the physician would ask her to do?

a. Serum potassium

b. CBC

c. Serum calcium

d. Serum chloride
HYPOGLYCEMIA AND HYPOCALCEMIA THEY GO TOGETHER
46. After the delivery of an infant of diabetic mother, blood glucose

of the infant was 60 mg/dl. Which other investigation does the

sister expects that the physician would ask her to do?

a. Serum potassium

b. CBC

c. Serum calcium

d. Serum chloride
IDM

• Most common congenital heart

• Most specific

• Most common neurological abnormality

• Most specific
IDM

• Most common congenital heart : VSD

• Most specific : TGA

• Most common neurological abnormality : NTD

• Most specific : CRS


47. Craniopagus is defined as fusion of?

AIIMS MAY 2018

a. Head and spine

b. Head only

c. Thorax and spine

d. Thorax only
47. Craniopagus is defined as fusion of?

AIIMS MAY 2018

a. Head and spine

b. Head only

c. Thorax and spine

d. Thorax only
48. A 4-year-old female came to the OPD with complaint of urinary

incontinence leading to continuous dribbling of urine. The image

below shows the pathology from which the patient is suffering,

most likely diagnosis is?

AIIMS MAY 2018

a. Gastroschisis

b. Bladder exstrophy

c. Superior vesicle fissure

d. Omphalocele
Superior vesicle fissure
48. A 4-year-old female came to the OPD with complaint of urinary

incontinence leading to continuous dribbling of urine. The image

below shows the pathology from which the patient is suffering,

most likely diagnosis is?

AIIMS MAY 2018

a. Gastroschisis

b. Bladder exstrophy

c. Superior vesicle fissure

d. Omphalocele
49: hyperglycemia in a neonate is defined as blood glucose

>___mg/dl ?

AIIMS 2017

1. 150

2. 125

3. 180

4. 100
49: hyperglycemia in a neonate is defined as blood glucose

>___mg/dl ?

AIIMS 2017

1. 150

2. 125

3. 180

4. 100
LAST QUESTION No.50
•Thank You
Q: A 7 year old with episodes of day dreaming in the class. He also
had staring episodes. What is the best management ?

A. Sodium valproate
B. Lamotrigine
C. Phenobarbitone
D. eptoin
Q: A 7 year old with episodes of day dreaming in the class. He also
had staring episodes. What is the best management ?

A. Sodium valproate
B. Lamotrigine
C. Phenobarbitone
D. eptoin
• Which is the absolute contraindication of breast feeding ?

A. Hepatitis A
B. Breast abscess
C. Galactosemia
D. Herpes Simplex
• Which is the absolute contraindication of breast feeding ?

A. Hepatitis A
B. Breast abscess
C. Galactosemia
D. Herpes Simplex
Contraindication of breast feeding
Contraindication of breast feeding

• Galactosemia

• Lactose intolerance

• Mother on Chemo + Radio

• Active untreated TB (relative)


Q: which of the following is not true about colostrum ?

a) Rich in IgA
b) Rich in proteins + amino acid
c) Rich in fat & sugar
d) Lemon yellow in colour
Q: which of the following is not true about colostrum ?

a) Rich in IgA
b) Rich in proteins + amino acid
c) Rich in fat & sugar
d) Lemon yellow in colour
v Colostrum :the first

immunization of the baby


v Colostrum :the first

immunization of the baby

v Golden thick yellow milk during

first 3 days of life


v IgA*, proteins*(as compared to

breast milk), WBCs

v ideal first MEAL for the baby


Remember HSP also it is IgA deposition everywhere
v Transitional milk :

Composition is in

between colostrum

and mature milk


v Mature milk: thin,

watery & contain less

proteins & more fat +

sugar are more **


Q: 8 year old child presented with hematuria with
history of sore throat since 2 weeks. On further
evaluation, there is cola colour urine. What Is the most
probable diagnosis ?

A. PSGN
B. Membranous nephropathy
C. IgA Nephropathy
D. Minimal Change Disease
Q: 8 year old child presented with hematuria with
history of sore throat since 2 weeks. On further
evaluation, there is cola colour urine. What Is the most
probable diagnosis ?

A. PSGN
B. Membranous nephropathy
C. IgA Nephropathy
D. Minimal Change Disease
Q: 8 year old child presented with hematuria with
history of sore throat since 2 weeks. On further
evaluation, there is cola colour urine. What Is the most
probable diagnosis ?

A. PSGN
B. Membranous nephropathy
C. IgA Nephropathy – short history of days
D. Minimal Change Disease
Q: At what level spinal cord ends in newborn & infants ?

A. L1
B. L2
C. L3
D. L4
Q: At what level spinal cord ends in newborn & infants ?

A. L1
B. L2
C. L3
D. L4
Q: At what level spinal cord ends in newborn & infants ?

A. L1 – CHILDREN
B. L2
C. L3
D. L4
Q: 5 year old child presented with fever with rash as seen in the
image. Which is the most probable diagnosis ?

A. 5th disease
B. 6th disease
C. Measles
D. Varicella
• Roseola infantum
• Roseola infantum

• CAUSED BY HHV 6 >7


• Roseola infantum

• CAUSED BY HHV 6 >7

• ALSO KNOWN AS 6TH DISEASE


• Roseola infantum

• CAUSED BY HHV 6 >7

• ALSO KNOWN AS 6TH DISEASE

• 5th disease: erythema infectiosum ( slapped cheek


appearance )
• Roseola infantum

• CAUSED BY HHV 6 >7

• ALSO KNOWN AS 6TH DISEASE

• 5th disease: erythema infectiosum ( slapped cheek


appearance )

• Nagayma spot is seen in roseola infantum


• Roseola infantum

• CAUSED BY HHV 6 >7

• ALSO KNOWN AS 6TH DISEASE

• 5th disease: erythema infectiosum ( slapped cheek


appearance )

• Nagayma spot is seen in roseola infantum

• In roseola infantum fever will subside then rash


will come
• Uvulo palato glossal junction
• Macules / ulcers
• Nagayama spots
Q: 5 year old child presented with fever with rash as seen in the
image. Which is the most probable diagnosis ?

A. 5th disease
B. 6th disease
C. Measles
D. Varicella
Q: A 4 year old baby with fever from last 5 days. On further
evaluation he had chest indrawing and refusal to feed. His RR was 46
/ min. what is the best possible diagnosis ?

A. No pneumonia
B. Severe pneumonia
C. Very severe pneumonia
D. Pneumonia
• Fast breathing cut off
• WHO criteria of fast breathing

• 60:

• >50:

• >40:

• >30:
• WHO criteria of fast breathing

• Respiratory rate >60: 0-2 months

• Respiratory rate >50: 3- 12 months

• Respiratory rate >40: 1-5 yr

• Respiratory rate >30: >5 yr


• New WHO ARI classification 2014:
• New WHO ARI classification 2014:

• Fast breathing and/or chest indrawing:

pneumonia
• New WHO ARI classification 2014:

• Fast breathing and/or chest indrawing:

pneumonia

• Danger sign or stridor in a calm child :

severe pneumonia.

• There is no very severe pneumonia


• PNEUMONIA:
• PNEUMONIA: AMOXICYLLIN 5 DAYS
• PNEUMONIA: AMOXICYLLIN 5 DAYS

• SP02 < 90% - REFER IMMEDIATELY


• PNEUMONIA: AMOXICYLLIN 5 DAYS

• SP02 < 90% - REFER IMMEDIATELY

• SEVERE PNEUMONIA: AMPI + GENTA

• REFER

Rest is very important *


What are the danger signs?

Mnemonic: CiLViUS
What are the danger signs?

Mnemonic: CiLViUS

C: convulsions

I: inability to drink or breast feed

L: lethargy
What are the danger signs?

Mnemonic: CiLViUS

V: vomiting

U: unconsciousness

S: Sleepiness
What are the danger signs?

Mnemonic: CiLViUS

C: convulsions

I: inability to drink or breast feed

L: lethargy

V: vomiting

U: unconsciousness

S: Sleepiness
Q: A 4 year old baby with fever from last 5 days. On further
evaluation he had chest indrawing and refusal to feed. His RR was 46
/ min. what is the best possible diagnosis ?

A. No pneumonia
B. Severe pneumonia
C. Very severe pneumonia
D. Pneumonia
Q: A 1 year old child came with 10-12 episodes of loose stools. He
also had hypokalemia. Even after the correction of hypokalemia
patient had features of muscle weakness + floppiness. Which is the
closest possible cause ?

A. Hypocalemia
B. Hypomagnesemia
C. Hypoalbuminemia
D. None of the above
Q: A 1 year old child came with 10-12 episodes of loose stools. He
also had hypokalemia. Even after the correction of hypokalemia
patient had features of muscle weakness + floppiness. Which is the
closest possible cause ?

A. Hypocalemia - tetany
B. Hypomagnesemia
C. Hypoalbuminemia
D. None of the above
Q: A 1 year old child came with 10-12 episodes of loose stools. He
also had hypokalemia. Even after the correction of hypokalemia
patient had features of muscle weakness + floppiness. Which is the
closest possible cause ?

A. Hypocalemia - tetany
B. Hypomagnesemia
C. Hypoalbuminemia
D. None of the above
• Life is a journey not a race
Q: A 3 year old boy with syndromic facies is having hypocalcemia
with microdeletion in chromosome number 22q11.2. what is the best
possible diagnosis

A. Turner syndrome
B. Patau syndrome
C. Digeorge syndrome
D. Down syndrome
DiGeorge Syndrome

22-
DiGeorge Syndrome

• C cardiac abnormality
DiGeorge Syndrome

• C cardiac abnormality

• A abnormal facies
DiGeorge Syndrome

• C cardiac abnormality

• A abnormal facies

• T thymic APLASIA
DiGeorge Syndrome

• C cardiac abnormality

• A abnormal facies

• T thymic APLASIA

• C cleft palate, cardiac outflow


trace defects
DiGeorge Syndrome

• C cardiac abnormality

• A abnormal facies

• T thymic APLASIA

• C cleft palate, cardiac outflow


trace defects

• H hypocalcemia ( due to
maldev. Of parathyroid
hormone )
DiGeorge Syndrome

• C cardiac abnormality

• A abnormal facies

• T thymic APLASIA

• C cleft palate, cardiac outflow


trace defects

• H hypocalcemia ( due to maldev.


Of parathyroid hormone )

• 22- 22q11 deletion chromosome


• Hypertelorism

• Anti mongoloid slant

• Low set ears

• Micrognathia

• Aortic arch abnormalities

• Cono truncal abn. – MC – truncus arteriosus


Q: A 3 year old boy with syndromic facies is having hypocalcemia
with microdeletion in chromosome number 22q11.2. what is the best
possible diagnosis

A. Turner syndrome
B. Patau syndrome
C. Digeorge syndrome
D. Down syndrome
Q: Which of the following embryological
development is affected in the following image ?

A. Anterior neuropore closure failure


B. Posterior neuropore closure failure
C. Whole neural tube is exposed
D. It is a forebrain anomaly
Q: Which of the following embryological development is
affected in the following image ?

A. Anterior neuropore closure failure – leads to cranial


defect
B. Posterior neuropore closure failure
C. Whole neural tube is exposed
D. It is a forebrain anomaly
Q: Which of the following embryological development is
affected in the following image ?

A. Anterior neuropore closure failure – leads to cranial


defect
B. Posterior neuropore closure failure – leads to caudal
defect
C. Whole neural tube is exposed
D. It is a forebrain anomaly
Q: sugar baby appearance / fatty baby ?

A. Marasmus
B. Kwashiorkor
C. Failure to thrive
D. Rickets
Q: sugar baby appearance / fatty baby ?

A. Marasmus
B. Kwashiorkor
C. Failure to thrive
D. Rickets
Q: 5 days old baby passing stool from the umbilicus, what is the
nearest possible explanation ?

A. Omaphalocele
B. Gastrochisis
C. Patent urachus
D. Patent Vitellointestinal duct
E. Ureterocele
There is a communication between umbilicus and bladder
Passing of urine from the umbilicus not stool
Q: 5 days old baby passing stool from the umbilicus, what is the
nearest possible explanation ?

A. Omaphalocele
B. Gastrochisis
C. Patent urachus
D. Patent Vitellointestinal duct / omphalomesentric duct
E. Ureterocele
Q: young child with the following rede cell indices with history of
recurrent blood transfusions

Hb: 4 gm/dl
MCV : 24 fl
MCH: 60 pg
MCHC 30%
A. Iron def anemia
B. Thalessemia
C. Sickle cell anemia
D. Megaloblastic anemia
A. Iron def anemia
B. Thalessemia
C. Sickle cell anemia
D. Megaloblastic anemia
Q: A 4 days old baby who is having distress which improves on crying.
What is your best possible diagnosis ?
Q: A 4 days old baby who is having distress which improves on crying.
What is your best possible diagnosis ?

A. Choanal atresia
B. TEF
C. HMD
D. Cleft lip and palate
Q: A 4 days old baby who is having distress which improves on crying.
What is your best possible diagnosis ?

A. Choanal atresia
B. TEF
C. HMD
D. Cleft lip and palate
The secret of your future is hidden in your daily routine
Q: a 10 year old child came with jaundice which was seen after
having stress / strenuous exercise. His liver enzymes were normal
and he was having unconjugated hyperbilirubinemia. What is your
best possible diagnosis ?

A. Dubin johnson syndrome


B. Rotor syndrome
C. Gilbert syndrome
D. Viral hepatiis
Q: a 10 year old child came with jaundice which was seen after
having stress / strenuous exercise. His liver enzymes were normal
and he was having unconjugated hyperbilirubinemia. What is your
best possible diagnosis ?

A. Dubin johnson syndrome


B. Rotor syndrome
C. Gilbert syndrome
D. Viral hepatiis
Q: Gum Bleeding is seen in which deficiency ?

A. Rickets
B. Scurvy
C. Megaloblastic anemia
D. Thalessemia
Q: Gum Bleeding is seen in which deficiency ?

A. Rickets
B. Scurvy
C. Megaloblastic anemia
D. Thalessemia
• People who hate me, motivate me !!
Q: 25 days old baby with chronic constipation cam with complaints of
abdominal distension. On further evaluation he had history of not
passing stool in first 3 days of life. what is the best management ?

A. Barium meal
B. Barium swallow
C. Rectal biopsy
D. Challenge test
HIRSCHSPRUNG DISEASE
• C/F: delayed passage of meconium f/b

bilious vomiting & abdominal distension


• C/F: delayed passage of meconium f/b

bilious vomiting & abdominal distension

• Enterocolitis (fever, bloody stools) is

usually seen b/w 2-4 weeks


• (Rectal) biopsy* - Absence of

ganglions*, nerve trunk hypertrophy &

inc. acetylcholinesterase activity in

involved segment
Q: 25 days old baby with chronic constipation cam with complaints of
abdominal distension. On further evaluation he had history of not
passing stool in first 3 days of life. what is the best management ?

A. Barium meal
B. Barium swallow
C. Rectal biopsy – MC area is rectum
D. Challenge test
Q: A 7 year old baby with rash in the lower limbs. He
also complaints of joint pain and hematuria. Although
his platelets count were normal. What is your best
possible diagnosis ?

A. ITP
B. HSP
C. Kawasaki
D. Wegners granulomatosis
Q: A 7 year old baby with rash in the lower limbs. He
also complaints of joint pain and hematuria. Although
his platelets count were normal. What is your best
possible diagnosis ?

A. ITP
B. HSP
C. Kawasaki
D. Wegners granulomatosis
• Definition: leukocytoclastic vasculitis &

IgA** deposition in small vessels of skin,


joints, GI tract, kidney
v Leukocytoclastic means : debris of neutrophils
in the blood vessels
• Skin- palpable purpura mainly on lower
limbs & buttocks*
• Joints- oligoarthritis <=4 / arthralgia in
75 % of cases
• GI Tract – in 80% seen – abdominal pain,
diarrhea & melena
• GI Tract – in 80% seen – abdominal pain,
diarrhea & melena

• Kidney: 50% - microscopic hematuria,


proteinuria, hypertension, nephritis,
nephrotic, renal failure 2%.
• GI Tract – in 80% seen – abdominal pain,
diarrhea & melena

• Kidney: 50% - microscopic hematuria,


proteinuria, hypertension, nephritis,
nephrotic, renal failure 2%.

• CNS – cerebral hemorrhage, seizures.


• Remember 3-10

• Rash stays for 3-10 days

• And is seen in age gp 3-10 yrs (90%)


• Diagnosis: diagnosis of HSP is CLINICAL

• Leukocytosis, thrombocytosis, normal


platelet count

• Occult blood in stool

• Albumin very low

• Biopsy- IgA deposition


• Rx: mild- supportive
• Rx: mild- supportive

• Prednisolone 1mg per kg per day for 1-2 weeks


reduces abdominal pain and joint pain
• Rx: mild- supportive

• Prednisolone 1mg per kg per day for 1-2 weeks reduces


abdominal pain and joint pain

• Prognosis: self limiting course lasting around 4 weeks


• Good prognosis
• Good prognosis

• 15-60% have recurrences

• 1-2 % will develop CKD chronic kid. Disease.


•Tetrad
•Palpable purpura
•Palpable purpura
•Arthritis
•Palpable purpura
•Arthritis
•Abdominal pain
•Palpable purpura
•Arthritis
•Abdominal pain
•Hematuria
Q21: A 6 year old child with abdominal pain &
red, elevated skin rashes, mainly involving the
lower limbs: what is your probable diagnosis:

NEET PG JAN 2019

A. Kawasaki

B. HSP

C. Varicella infection
D. meningococcemia

Ref: Nelson 20th p 1768,ghai 8/e p11 b


Q: what is the first line of drug for febrile seizures?

A. Diazepam
B. Midazolam
C. phenytoin
D. Phenobarbitone
Q: what is the first line of drug for febrile seizures?

A. Diazepam
B. Midazolam
C. phenytoin
D. Phenobarbitone
Q: what is the first line of drug for febrile seizures?

A. Diazepam
B. Midazolam
C. phenytoin – 2nd line
D. Phenobarbitone
• Don’t be bitter by past, be better by past
Q: Menkes Disease is associated with which metabolism ?

A. Copper
B. Molybednum
C. Zinc
D. Calcium
Q: Menkes Disease is associated with which metabolism ?

A. Copper
B. Molybednum
C. Zinc
D. Calcium
•Menkes
Disease
• Mutation: _____ ?
• Mutation: ATP7A
• Mutation: ATP7A

• Mutation in the gene encoding

copper
• Copper level in the brain & liver

are low and inc. level in

enterocytes & fibroblasts


• Delayed dentition

• They die before 3 yrs


• MCC death: cardiac

rupture
• Loss of previously attained

developmental milestones
• Boys +
• Wormion bones
• Multiple # , flaring of ribs

• Battered baby syndrome is

one of the DD
Trichorrhexis nodosa: defect in the
hair shaft characterized by thickening
or weak points (nodes)
that cause the hair to
break off easily.
• Pathognomic: Pili torti is a rare hair condition
characterized by fragile hair.
• 180 degree shifting of the hair shaft.
• Cutis Laxa: In which the skin hangs loosely in folds.
Patients develop a prematurely aged appearance.
Q: which is the best possible finding seen in infant of diabetic mother ?

A. Hyperglycemia
B. Hypoglycemia
C. Hyperthermia
D. IUGR
Q: which is the best possible finding seen in infant of diabetic mother ?

A. Hyperglycemia
B. Hypoglycemia
C. Hyperthermia
D. IUGR
WHAT HAPPENS IN MOTHER
?

• Insulin is secreted by ?

• What is the job of insulin ?


• Insulin &
glucagon is
secreted by
beta cells of
pancreas
• Insulin &
glucagon is
secreted by beta
cells of pancreas
• Insulin job is to
convert glucose
into glycogen
(HAPPENS IN
LIVER CELLS)
• No insulin,
no
conversion,
more and
more glucose
In Mother
• In the liver cells glucose is
converted into glycogen
• If insulin deficiency is there
glucose will not enter in the
liver cells and more and more
glucose will present is in the
blood
• Pancreas releases GLUCAGON when the

concentration of insulin is too low.

• Glucagon causes conversion of Glycogen into

glucose.
Infant of diabetic mother Pederson’s
hypothesis

• Maternal diabetes (hyperglycemia)


Infant of diabetic mother Pederson’s
hypothesis

• Maternal diabetes (hyperglycemia)

• à fetal hyperglycemia (through placenta &


fetal circulation)
• à fetal pancreas stimulated to produce
excessive insulin

• à neonatal hypoglycemia
Q: what will be the dose of folic acid for a mother who had a
previous baby with neural tube defect ?

A. 400 microgram –
B. 4000 microgram –
C. 1 mg
D. 0.1 mg
Q: what will be the dose of folic acid for a mother who had a
previous baby with neural tube defect ?

A. 400 microgram – usual dose 0.4 mg


B. 4000 microgram – 2nd dose 4 mg
C. 1 mg
D. 0.1 mg
Q: Dystrophin gene is absent in

A. DMD
B. Myasthenia gravis
C. Rickets
D. None of the above
Q: Dystrophin gene is absent in

A. DMD
B. Myasthenia gravis
C. Rickets
D. None of the above
Muscular Dystrophy
Muscular Dystrophy
• (DMD) Duchenne muscular dystrophy
Muscular Dystrophy
• (DMD) Duchenne muscular dystrophy
• Becker muscular dystrophy
Muscular Dystrophy
• (DMD) Duchenne muscular dystrophy
• Becker muscular dystrophy
• Fascioscapulohumeral muscular dystrophy
(landouzy dejerine disease )
DMD-Duchenne Muscular Dystrophy

• It is the most common hereditary neuromuscular disease


DMD-Duchenne Muscular Dystrophy

• It is the most common hereditary neuromuscular disease

• Genetics: X linked recessive: Xp21 (dystrophin gene) is


affected
DMD-Duchenne Muscular Dystrophy

• It is the most common hereditary neuromuscular disease

• Genetics: X linked recessive: Xp21 (dystrophin gene) is


affected

• P short arm

• Q long arm
DMD-Duchenne Muscular Dystrophy

• It is the most common hereditary neuromuscular disease

• Genetics: X linked recessive: Xp21 (dystrophin gene) is


affected

• P short arm

• Q long arm

• C/F: Progressives weakness, developmental delay,


pseudohypertrophy of calf muscles & wasting of thigh
muscles.
DMD-Duchenne Muscular Dystrophy

• It is the most common hereditary neuromuscular disease

• Genetics: X linked recessive: Xp21 (dystrophin gene) is


affected

• P short arm

• Q long arm

• C/F: Progressives weakness, developmental delay,


pseudohypertrophy of calf muscles & wasting of thigh
muscles.

• After calves next common site is – ?


DMD-Duchenne Muscular Dystrophy

• It is the most common hereditary neuromuscular disease

• Genetics: X linked recessive: Xp21 (dystrophin gene) is


affected

• P short arm

• Q long arm

• C/F: Progressives weakness, developmental delay,


pseudohypertrophy of calf muscles & wasting of thigh
muscles.

• After calves next common site is – tongue


DMD-Duchenne Muscular Dystrophy

• It is the most common hereditary neuromuscular disease

• Genetics: X linked recessive: Xp21 (dystrophin gene) is affected

• P short arm

• Q long arm

• C/F: Progressives weakness, developmental delay,


pseudohypertrophy of calf muscles & wasting of thigh muscles.

• After calves next common site is – tongue

• Gower Sign: evident by 3, fully expressed by 5-6 yrs.

• A Trendelenburg gait appears at this time


Gower sign:

• Indicates weakness of proximal muscles

• Patient uses hands and arms to walk up from squatting position


due to lack of hip and thigh muscles strength
Gene mutation in DMD ?
• Gene mutation in DMD ?
• Xp21 dystrophin gene
Is it proximal or distal myopathy?
Is it proximal or distal myopathy?
Ans: proximal
Most common cause of death in DMD?
• Most common cause of death in DMD?
• Respiratory failure during sleep
• Which is more severe ?
• DMD or Beckers ?
• Which is more severe ?
• DMD
• Early onset and more severe
Test Name Obtained value Units Bio. Ref. Intervals Method
(Age/Gender specific)

*Creatinine Kinase (CPK)- Serum 14402 U/L 30-200 NADP, NAC (N

acetyl L cysteine)
Comments:
• Measurements of creatinine kinase are used in the diagnosis and treatment of myocardial infarction and muscle
disease
• Creatinine kinase may also be elevated in muscle injury or strenuous exercise

Correlate Clinically Result rechecked and verified for abnormal


cases

***End of Report***
• Diagnosis: Serum creatine kinase ( CK) is
greatly elevated (N- <160 IU/L) 15-35,000 IU/L
• Diagnosis: Serum creatine kinase ( CK) is
greatly elevated (N- <160 IU/L) 15-35,000 IU/L

• PCR of dystrophin gene confirms the diagnosis


• Diagnosis: Serum creatine kinase ( CK) is
greatly elevated (N- <160 IU/L) 15-35,000 IU/L

• PCR of dystrophin gene confirms the diagnosis

• If PCR is normal, muscle biopsy with


dystrophin immunocytochemistry is done
NEET
last 5 year questions
Dr.anand
PEDIATRICS
BY
DR.ANAND
M.D PEDIATRICS
• MBBS
• MBBS
• MD
• MBBS
• MD
• Assistant Professor
• MBBS
• MD
• Assistant Professor
• Teaching * >7 years
• UG
• PG
• DCH
• MD
• DM
• MRCPCH
PEDIATRICS
BY
DR.ANAND
M.D PEDIATRICS
ENTIRE WATER OF THE SEA
CANT SINK UNLESS IT GETS
INSIDE THE SHIP
NEGATIVITY OF THE WORLD
CANT PUT YOU DOWN UNLESS
YOU ALLOW IT TO GET INSIDE
YOU
•Work hard in
silence
•Work hard in
silence
•Lets success
makes the noise
•NEET PG
•2022
QUESTIONS
Q: a 4 year old with white marks with foamy appearance on the
sclera of the child. Which of the following symptom is associated
with the lesion mentioned above ?

A. Angular stomatitis

B. Deafness

C. Glossitis

D. Conjunctival xerosis
Q: a 4 year old with white marks with foamy appearance on the
sclera of the child. Which of the following symptom is associated
with the lesion mentioned above ?

A. Angular stomatitis

B. Deafness

C. Glossitis

D. Conjunctival xerosis
X1A

X1B

X2

X3A

X3B
X1A Conjunctival
xerosis
X1B

X2

X3A

X3B
X1A Conjunctival
xerosis
X1B Bitot spots

X2

X3A

X3B
X1A Conjunctival
xerosis
X1B Bitot spots

X2 corneal xerosis

X3A

X3B
X1A Conjunctival
xerosis (EARLIEST
SIGN)**
X1B Bitot spots

X2 Corneal
xerosis(IAP**)
X3A Corneal ulceration
in <1/3rd of corna

X3B Corneal ulceration


in >1/3rd of corna
XN

XS

XF
XN NIGHT BLINDNESS.
(EARLIEST
SYMPTOM**)

XS SCARRING

XF FUNDAL CHANGES
X1A Conjunctival
xerosis (EARLIEST
SIGN)**
X1B Bitot spots

X2 Corneal xerosis

X3A Corneal ulceration


in <1/3rd of corna

X3B Corneal ulceration


in >1/3rd of corna
XN NIGHT BLINDNESS.
(EARLIEST
SYMPTOM**)

XS SCARRING

XF FUNDAL CHANGES
Q: Rash over the neck +
h/o child on Maize diet ?
Q: Rash over the neck +
h/o child on Maize diet ?

A.Ribloflavin
B.Thiamine
C.Folic acid
D.Niacin
Q: Rash over the neck +
h/o child on Maize diet ?

A.Ribloflavin
B.Thiamine
C.Folic acid
D.Niacin
•Maize has no Niacin
Q: Rash over the neck +
h/o child on Maize diet ?

A.Ribloflavin
B.Thiamine
C.Folic acid
D.Niacin
Q: Rash over the neck +
h/o child on Maize diet ?

A.Ribloflavin
B.Thiamine
C.Folic acid
D.Niacin
Q: Rash over the neck + h/o
child on Maize diet ?

A.Ribloflavin – angular
stomatitis
B.Thiamine
C.Folic acid
D.Niacin
Q: Rash over the neck + h/o
child on Maize diet ?

A.Ribloflavin – angular
stomatitis, CHELTIIS
B.Thiamine – BERI BERI
C.Folic acid
D.Niacin
Q: Rash over the neck + h/o
child on Maize diet ?

A.Ribloflavin – angular
stomatitis, CHELTIIS
B.Thiamine – BERI BERI
C.Folic acid –
MEGALOBLASTIC ANEMIA
D.Niacin
Q: A 4 year old child with
seizures. What is the first
line of management
pertaining to “D” – TABCD
A.Disability
B.Distraction
C.Diarrhea
D.Dyspnea
Q: A 4 year old child with
seizures. What is the first
line of management
pertaining to “D” – TABCD
A.Disability
B.Distraction
C.Diarrhea
D.Dyspnea
Q: A 4 year old child with
cough, fast breathing had
following x Ray finding?
What is the next best step
in management ?
A. Steroids + antibiotics
B. Steroids + antibiotics + adrenaline
C. Steroids only
D. Steroids + adrenaline
A. Steroids + antibiotics
B. Steroids + antibiotics + adrenaline
C. Steroids only
D. Steroids + adrenaline
Q: which is the most common seizure in a
newborn ?
A. Hypoxic seizure
B. Absence seizure
C. Febrile seizure
D. None of the above
Q: which is the most common seizure in a
newborn ?
A. Hypoxic seizure
B. Absence seizure
C. Febrile seizure
D. None of the above
Q: which is the most common seizure in a
newborn ?
A. Hypoxic seizure
B. Absence seizure
C. Febrile seizure (6m – 5 yr )
D. None of the above
Q: which is the most commonest cause of the
disease shown below ?
A. Picorna virus
B. Coxsackie virus
C. Para influenza virus
D. None of the above
Q: which is the most commonest cause of the
disease shown below ?
A. Picorna virus
B. Coxsackie virus A 16 & enterovirus
C. Para influenza virus
D. None of the above
•Steeple sign is
seen in ?
Q: a 10 year old female child with loose stool with pain
abdomen with failure to thrice with HLA DQ2 positive.
What is the best treatment which can fit in the case
scenario ?
A. Gluten free diet
B. Low lactose diet
C. Lactose free diet
D. None of the above
•Celiac Disease
•Celiac Disease

•It is a disease of g___ & g___


•Celiac Disease

•It is a disease of genes & grains


•Celiac Disease

•It is a disease of
•Genes
•Grains
•Celiac Disease

•It is a disease of
•Genes – HLA class 2
•Grains
•Celiac Disease

•It is a disease of
•Genes – HLA class 2
•Grains – Gluten
•Patients with CD have a
variant of HLA genes

•HLA DQ2*
•HLA DQ8*
•Patients with CD have a
variant of HLA genes

•HLA DQ2
•HLA DQ8
•Human leukocyte antigen
• Resulting in the hallmark lesions ?
• Villus atrophy
• Crypt hyperplasia
• increased intraepithelial lymphocytes
(IELs)
Pediatric surgery
•Patient is sensitive to ?
Pediatric surgery
•Patient is sensitive to

•Gluten

•Where all gluten is present ?


Wheat
• Whole wheat Rotis
• Clinical features

•.
•Clinical features

•Typical

•Atypical
•Typical
•Typical
•Typical
•Typical
•Typical:
•Typical: FTT
• Which is more common in a patient ?

A.Intestinal manifestations
B. Extra intestinal manifestations
• Which is more common in a patient ?

A.Intestinal manifestations
B. Extra intestinal manifestations
Intense itchy blisters filled with water
• Dermatitis herpetiformis
• Clusture of itchy bumps
• Which looks like eczema
• Most common location ?
• Elbows / knees / lower back
•Enamel hypoplasia
•Iron def. anemia resistant to oral
iron therapy
•Short stature
•Delayed puberty
• Arthritis

• Osteopenia
• CNS ?
• CNS ataxia / epilepsy
Pediatric surgery
• Most common extra intestinal manifestation of
celiac disease
Pediatric surgery
• Most common extra intestinal manifestation of
celiac disease: Iron def. Anemia
Pediatric surgery
• Most common extra intestinal manifestation of
celiac disease
• GLUTEN SENSITIVITY
Pediatric surgery
• Most common extra intestinal manifestation of
celiac disease
• GLUTEN SENSITIVITY
• Which is the MCC OF CHRONIC DIARRHEA IN
CHILDREN ?
Pediatric surgery
• Most common extra intestinal manifestation of
celiac disease
• GLUTEN SENSITIVITY
• MCC OF CHRONIC DIARRHEA IN CHILDREN
• SMALL BOWEL / LARGE BOWEL DIARRHEA ?
Pediatric surgery
• Most common extra intestinal manifestation of
celiac disease
• GLUTEN SENSITIVITY
• MCC OF CHRONIC DIARRHEA IN CHILDREN
• SMALL BOWEL DIARRHEA
Pediatric surgery
• Most common extra intestinal manifestation of
celiac disease
• GLUTEN SENSITIVITY
• MCC OF CHRONIC DIARRHEA IN CHILDREN
• SMALL BOWEL DIARRHEA
• GROWTH FAILURE
Pediatric surgery
• Most common extra intestinal manifestation of
celiac disease
• GLUTEN SENSITIVITY
• MCC OF CHRONIC DIARRHEA IN CHILDREN
• SMALL BOWEL DIARRHEA
• GROWTH FAILURE
• ANEMIA
• Which is the first serological testing done for celiac
disease ?

A. tTGA

B. EMA

C. Deamidated gliadin peptide


• Which is the first serological testing done for celiac
disease ?

A. tTGA – Ig A tissue transglutaminase

B. EMA

C. Deamidated gliadin peptide


• If results of tTga is equivocal ? Then ?

A. tTGA – Ig A tissue transglutaminase

B. EMA

C. Deamidated gliadin peptide


• If results of tTga is equivocal ? Then ?

A. tTGA – Ig A tissue transglutaminase

B. EMA – endomyseal antibodies

C. Deamidated gliadin peptide


• If the child is <2 years ? then ?

A. tTGA – Ig A tissue transglutaminase

B. EMA – endomyseal antibodies

C. Deamidated gliadin peptide


• If the child is <2 years ? then ?

A. tTGA – Ig A tissue transglutaminase

B. EMA – endomyseal antibodies

C. Deamidated gliadin peptide


•Boot shape
heart is seen
in ?
•TOF: Tetralogy
Of Fallot
•Q: Adult person
with borderline
hypertension with
with x ray showing:
•E sign
•Reverse 3 sign
•COA
Q: Which one of the following is an Oligemic lung field condition ?

A. ASD
B. VSD
C. PDA
D. EBSTEIN ANOMALY
Gastroschisis
• Always Covered by the sac (peritoneum)
• Contents: Liver + Spleen
Q: Baby with
hypercalcemia + supra
valvular aortic
A. Down syndrome
stenosis +
typical facial features ?
B. Noonan syndrome
C. William syndrome
D. Wiskott Aldrich syndrome
• An elf (plural: elves) is a
type of human-shaped
supernatural being in
German mythology
• William syndrome – round face, long philtrum short

upturned nose (elfin facies)

• supra valvular aortic stenosis*

• Inc. calcium
Q: 5 yr old Child with Rash (eczema) +
Thrombocytopenia + Recurrent sinopulmonary
infection ?
Q: 5 yr old Child with Rash (eczema) +
Thrombocytopenia + Recurrent sinopulmonary
infection ?

A. Down syndrome
B. Noonan syndrome
C. William syndrome
D. Wiskott Aldrich syndrome
Q: 5 yr old Child with Rash (eczema) +
Thrombocytopenia + Recurrent sinopulmonary
infection ?

A. Down syndrome
B. Noonan syndrome
C. William syndrome
D. Wiskott Aldrich syndrome
•Wiskott Aldrich
Syndrome
•Wiskott Aldrich Syndrome:
suspected with any boy who is
unusally bleeding/bruising with
thrombocytopenia
•Wiskott Aldrich Syndrome:
inheritance ?
•Wiskott Aldrich Syndrome: X
linked disease (males)
•Wiskott Aldrich Syndrome:
mutation in the ___ gene
•Wiskott Aldrich Syndrome:
mutation in the WASP gene
•Wiskott Aldrich Syndrome:
affected male baby comes
with bleeding
•Wiskott Aldrich Syndrome:
many do not survive
childhood
• After taking whiskey

• Ethyl Alcohol goes up

• So IgE and IgA are up

• And Mental function Goes down


• So IgM decreased with normal IgG
•Wiskott Aldrich Syndrome:
IgM is low, IgA + IgG is
normal, IgE is raised
Q: Green black stool of the new born is
due to ?

A. Meconium
B. Bile
C. Gut flora
Q: Green black stool of the new born is due to ?

A. Meconium
B. Bile
C. Biliverdin
D. Gut flora
•Q: newborn with
bilirubin 15 mg/dl
@ 5 days of life:
A. Exchange transfusion
B. Phototherapy
C. Stop breastfeeding
D. Do not give
phototherapy
•Q: newborn with
bilirubin 15 mg/dl
@ 5 days of life:
A. Exchange transfusion
B. Phototherapy
C. Stop breastfeeding
D. Do not give
phototherapy
•Indication of

exchange transfusion

are:
•Indication of exchange

transfusion are:

•Cord hemoglobin <10

g/dl or cord bilirubin >5

mg/dl
•Indication of exchange

transfusion are:

•DCT +ve

•H/O kernicterus or

Erythroblastosis in sibling
Q: Fever >5 days, conjunctivitis, polymorphous rash +
cervical adenopathy ?

A. Measles

B. Kawasaki
C. Mumps

D. HSP
•Kawasaki
•Introduction
•Which vessel is involved ?
•Do you see in Adults ?
•Diagnostic Criteria
•Also known as ?
•Also known as
mucocutaneous lymph node
syndrome
Kawasaki Disease

•Autoimmune disease
Kawasaki Disease

•Autoimmune disease

•Small / large vessel ?


Kawasaki Disease

•Autoimmune disease

•Medium vessel
Kawasaki Disease

•Autoimmune disease

•Medium vessel

•seen in adults ?
Kawasaki Disease

•Autoimmune disease

•Medium vessel

•seen in adults ?

•80% are <4 years


Kawasaki Disease

•Autoimmune disease

•Medium vessel

•Not seen in adults

•80% are <4 years


Kawasaki Disease

• Diagnostic criteria: Essential

Criteria:

• Fever for at least 5 days with any

4 out of 5

• Additional criteria: REEMA

features
R
E
E
M
A
4 OUT OF 5

•R – rash

(polymorphous

non vesicular )
4 OUT OF 5

•R – rash

(polymorphous non

vesicular )

•E – edema (hands,

feet)
4 OUT OF 5
• R – rash (polymorphous

non vesicular )

• E – edema (hands, feet)

• E – conjunctivitis (non

purulent)
4 OUT OF 5
• R – rash (polymorphous non

vesicular )

• E – edema (hands, feet)

• E – conjunctivitis (non purulent)

• M – Mucosal involvement

(strawberry tongue, cracked

lips)
4 OUT OF 5
• R – rash (polymorphous non
vesicular )

• E – edema (hands, feet)

• E – conjunctivitis (non purulent)

• M – Mucosal involvement
(strawberry tongue, cracked lips)

• A - adenopathy (cervical/unilateral)
+ non suppurative, non tender
Q: Fever >5 days, conjunctivitis, polymorphous rash +
cervical adenopathy ?

A. Measles

B. Kawasaki
C. Mumps

D. HSP
Q: Tall height + long
limb & digits + pedigree
chart ?
Q: Tall height + long limb
& digits + pedigree chart ?

•Marfan syndrome
ARACHNODACTYLY
•M
•A
•R
•F
•A
•N
•S
• M – Mitral valve prolapse
• A– arachnodactyly + aut.dom.
• R – retinal detachment
• F- fibrillin deficiency FBN1 ch.15q21
• A- aortic aneurysm
• N- -ve nitroprusside test
• S- supero lateral subluxation of lens
•HIM
•HIM
•Homocysteinuria
•Infero medial
subluxation
•False statement ?
Q:2 year old + RR 38 cpm
+ chest retractions – what
is our next step –

A. Refer
B. Conservative management
C. Antibiotics + Refer
D. Home Management
•New WHO ARI

classification 2014:
•New WHO ARI

classification 2014:

•Fast breathing and/or

chest indrawing:

pneumonia
• New WHO ARI classification

2014:

• Fast breathing and/or chest

indrawing: pneumonia

• Danger sign: severe pneumonia

or very severe pneumonia


•PNEUMONIA:
•PNEUMONIA:
AMOXICYLLIN 5 DAYS
•PNEUMONIA: AMOXICYLLIN
5 DAYS
•SP02 < 90% - REFER
IMMEDIATELY
• PNEUMONIA: AMOXICYLLIN 5 DAYS
• SP02 < 90% - REFER IMMEDIATELY
• SEVERE PNEUMONIA: AMPI + GENTA
• REFER
Q:2 year old + RR 38 cpm
+ chest retractions – what
is our next step –

A. Refer
B. No management
C. Antibiotics + Refer
D. Home Management
Q: 10 yr old boy with fatigue & abdominal pain after playing
football. On detailed history there was also painful swelling of
digits in the past. USG: small spleen; most likely diagnosis ?

A. Acute pancreatitis
B. Sickle cell anemia
C. Iron def. anemia
D. Thalessemia
•Sickle cell anemia
points:
•Sickle cell anemia
points:
•GO VAL
•Sickle cell anemia
points:
•GO VAL
•Glutamate is replaced
by valine ( amino acid)
• Due to this there will be an abnormal hemoglobin
• Due to this there will be an abnormal hemoglobin

• In the event of hypoxia – the hb polymerizes &


cause the red cells to change in shape and become
sickle cells
•Sickle cell anemia
points:
•Change in shape of
RBC
•Sickle cell anemia
points:
•Change in shape of RBC
•Gets destroyed in
Spleen
•Sickle cell anemia points:
•Change in shape of RBC
•Gets destroyed in Spleen

•Anemia
•Sickle cell anemia
points:
•Due to sickling of RBC
•Sickle cell anemia
points:
•Due to sickling of RBC

•There is deposition of
sickle shaped plaques
•Sickle cell anemia points:
•Dactylitis / bone pain /
stroke / retinopathy /
hand foot syndrome /
auto splenectomy / acute
chest syndrome
•Get a peripheral
smear:
•Get a peripheral
smear:
•Anisopoikilocytosis
•Get a peripheral
smear:
•Anisopoikilocytosis
•Retic count ___.
(more hemolysis)
•Get a peripheral
smear:
•Anisopoikilocytosis
•Retic count inc. (more
hemolysis)
•Get a peripheral smear:
•Anisopoikilocytosis
•Retic count inc. (more
hemolysis)
•Polychromasia
•Get a peripheral smear:
•Anisopoikilocytosis
•Retic count inc. (more
hemolysis)
•Polychromasia
•Inc. target cells
•Get a peripheral smear:
•Anisopoikilocytosis
•Retic count inc. (more
hemolysis)
•Polychromasia
•Inc. target cells
•Sickling test
•Get a peripheral smear:
•Anisopoikilocytosis
•Retic count inc. (more
hemolysis)
•Polychromasia
•Inc. target cells
•Sickling test
•Spleen biopsy: ?
•Get a peripheral smear:
•Anisopoikilocytosis
•Retic count inc. (more hemolysis)
•Polychromasia
•Inc. target cells
•Sickling test
•Spleen biopsy:
gamma gandy bodies
•Inv of choice ?
•Inv of choice ?
•HPLC
•High performance
liquid
chromatography
•If I am having sickle cell
anemia I will not have
______?
•If I am having sickle cell
anemia I will not have
F.malaria due to homozygous
sickle trait
•Sickle cell anemia is not
seen till _______ of life
•Sickle cell anemia is not
seen till 6 months of life
•Fetal hb prevents sickling
of RBC
•Hydroxyurea may prevent
the sickling
•Crew cut appearance
• Which syndrome is associated as a
complication of sickle cell anemia ?
• Which syndrome is associated as a
complication of sickle cell anemia ?

• Acute chest syndrome


• Which syndrome is associated as a
complication of sickle cell anemia ?

• Acute chest syndrome

• Chest pain + fever + cough


• Which syndrome is associated as a
complication of sickle cell anemia ?

• Acute chest syndrome

• Chest pain + fever + cough


• Which syndrome is associated as a
complication of sickle cell anemia ?

• Acute chest syndrome

• Chest pain + fever + cough

• Due to sickling in the blood vessels you will


see an emboli / infarction in the lungs
• Rx ?
• Rx ?

• High flow oxygen** (1st line)


• Rx ?

• High flow oxygen


• Analgesics
• IV antibiotics
• Rx ?

• High flow oxygen


• Analgesics
• IV antibiotics

• PRBC transfusion, fluids


Functional Anatomical
closure closure

Ductus venosus

Formane ovale

Ductus
arteriosus
Functional Anatomical
closure closure

Ductus venosus After removal 7 days


of placenta

Formane ovale

Ductus
arteriosus
Functional Anatomical
closure closure

Ductus venosus After removal 7 days


of placenta

Formane ovale During breast Months – Years


feeding

Ductus
arteriosus
Functional Anatomical
closure closure

Ductus venosus After removal 7 days


of placenta

Formane ovale During breast Months – Years


feeding

Ductus 10-15 hours 10-21 days


arteriosus
Q: Paramyxo virus causing
macular like whitish spot
opposite to molar teeth in a
child. Identify ?
•Koplik spots: discrete
red lesions with bluish
white spots in the
center on the inner
aspects of the cheek at
the level of 1st & 2nd molar
•Remember 3 c
•Remember 3 c
•Cough
•Coryza
•Conjuctivitis
•Cause:
•Cause: RNA virus-
family
paramyxoviridae
•Incubation period:
•Incubation period:
8-12 days
•Period of
communicability:
•Pathology:

•c/f:
•____ spots:
• Period of communicability: -
3+5 (Rash)
• Pathology:

• c/f:
• ____ spots:
•Pathology:

•(pathognomic for
measles)
•Pathology: Warthin
finkeldey giant cells
(pathognomic for
measles)
•Pathology: Warthin
finkeldey giant cells
(pathognomic for
measles)
•c/f: fever, coryza,
cough, conjunctivitis
A.Pre existing DM
B.Gestational DM
C.ACE inhibitor
D.Valproate
A.Pre existing DM
B.Gestational DM
C.ACE inhibitor
D.Valproate
•Sirenomelia
•Sirenomelia
•Caudal regression
•Sirenomelia
•Caudal regression:
Maternal DM
•Fusion of the
lower
extremities
•Claw sign or coiled spring sign
•Intussusception
Q: Vitamin B 12 is found in ?

A.Green leafy vegetables


B. Fruits
C. Vegetables
D.Animal products
Q: Vitamin B 12 is found in ?

A.Green leafy vegetables


B. Fruits
C. Vegetables
D.Animal products
•I
•F
•B
•BANK
•IRON: Duodenum
•FOLIC ACID: Jejunum
•B12: Ileum
•BANK
•Which chemical
compound comprise of
surfactant ?
Dipalmitoyl
Phosphatidylcholine(
lecithin)
Phosphatidylglycerol
Dipalmit
oyl
Phospha
tidylch…
Q: Which type of cell is
defective in surfactant ?
If you don’t sacrifice
for “what you want”
“what you want”
becomes the sacrifice
•NEET PG 2019
QUESTIONS
1) Bidextrous approach comes at what age ?

NEET PG 2019
1) Bidextrous approach comes at what age ?
4 MONTHS
NEET PG 2019
BMU
456
Q2: What is the interpretation of APGAR SCORE 3:

NEET PG 2019
A. Moderately depressed baby

B. Severely depressed baby

C. Normal

D. Mild birth asphyxia


Q2: What is the interpretation of APGAR SCORE 3:

NEET PG 2019
A. Moderately depressed baby

B. Severely depressed baby

C. Normal

D. Mild birth asphyxia


Q3: Where to look for pre ductal oxygen
saturation in PDA in a 3 minute born infant ?

NEET 2019 Question


1. Left upper limb

2. Left lower limb

3. Right upper limb

4. Right lower limb


Q3: Where to look for pre ductal oxygen
saturation in PDA in a 3 minute born infant ?

NEET 2019 Question


1. Left upper limb

2. Left lower limb

3. Right upper limb

4. Right lower limb


Q4: Which of the following is seen in CYSTIC FIBROSIS:

NEET PG JANUARY 2019

A. Low sweat chloride level

B. Elevated sweat chloride level

C. Low sweat sodium

D. Elevated sweat potassium level


•Cystic fibrosis
• Introduction

• Mutation

• Defect
Cystic Fibrosis

• Autosomal recessive
Cystic Fibrosis

• Autosomal recessive + pompe disease + acc. Of


glycogen in lysosome
Cystic Fibrosis

• Autosomal recessive

• Mutation in chromosome 7
Cystic Fibrosis

• Autosomal recessive

• Mutation in chromosome 7

• Primary defect: CFTR gene


Cystic Fibrosis

• Autosomal recessive

• Mutation in chromosome 7

• Primary defect: CFTR gene

• Other defects: chloride channels*, potassium


channels*, epithelial sodium channels*.
• CFTR: cystic fibrosis transmembrane conductance
regulator gene on chromosome 7q31.2
• Sweat ducts normally absorb sodium from
lumen which is decrease in CF causing salty
sweat
• ( high sodium and chloride) HALLMARK OF CF
• Sweat chloride level is >60mEq/L in cystic fibrosis
• Airways, pancreas & GIT: Normally sodium uptake is
thr from luminal fluid
• In CF there Is increase sodium and water uptake
from lumen
• Causing thick, viscid secretions
Fat and protein are not absorbed
• False +ve sweat chloride test:
• False +ve sweat chloride test:

• Eczema

• Ectodermal dysplasia
• False +ve sweat chloride test:

• Eczema

• Ectodermal dysplasia

• Malnutrition

• Congenital adrenal hyperplasia

• Adrenal insufficiency
• c/f:
• c/f: Resp System
•Repeated URTI
•Chronic cough *
•Sputum *
•Bronchiectasis *
•Atelectasis *
•Bronchiectasis *
•Atelectasis *
•Nasal polyp *
• H.influenzae, S.aureus are the first

organisms recovered from lung

secretions f/b p.aeruginosa*


• Mortality in cystic fibrosis is due to
• Mortality in cystic fibrosis is due to

• BURKHOLDERIA CEPACIA
•Genito urinary
•Genito urinary
•Azoospermia *
•Genito urinary
•Azoospermia *
•Infertility *
•Cong. b/l absence /
obliteration of vas deferense*
• Intestinal
• Meconium ileus** MOST COMMON

COMPLI. @ BIRTH

If sweat chloride are normal,


go for nasal transmembrane potential difference*
• Meconium ileus** MOST COMMON

COMPLI. @ BIRTH

• Distal intestinal obstruction syndrome

(DIOS)

If sweat chloride are normal,


go for nasal transmembrane potential difference*
• Meconium ileus** MOST COMMON

COMPLI. @ BIRTH

• Distal intestinal obstruction syndrome

(DIOS)

• Rectal prolapse

If sweat chloride are normal,


go for nasal transmembrane potential difference*
• Pancreatic:
• Pancreatic: exocrine insufficiency – will

lead to FOUL SMELLING STOOL

• Chronic pancreatitis
• Pancreatic: exocrine insufficiency – will

lead to FOUL SMELLING STOOL

• Chronic pancreatitis

If sweat chloride are normal,


go for nasal transmembrane potential difference*
•Hepatic:
•Hepatic:
•Focal biliary cirrhosis
•Hepatic:
•Focal biliary cirrhosis

•Prolonged neonatal jaundice


•Nutritional:
•Nutritional:
•FTT
•Nutritional:

•FTT

•Hypoproteinemia + edema

•Fat soluble vitamin def.


•Salt loss syndrome:
•Salt loss syndrome:
•Chronic metabolic alkalosis*
• Medical management:
• Medical management:

• Resp:
• Medical management:

• Resp:
• Bronchodilators
• Medical management:

• Resp:
• Bronchodilators
• Hypertonic saline
• Medical management:

• Resp:
• Bronchodilators
• Hypertonic saline
• Antibiotics
• Medical management:

• Resp:
• Bronchodilators
• Hypertonic saline
• Antibiotics
•Nutrtional:
•Nutrtional:
•High caloric diet
•Nutrtional:
•High caloric diet
•Pancreatic enzyme replacement
•Nutrtional:
•High caloric diet
•Pancreatic enzyme replacement
•Fat soluble vitamins A D E K
• Medical management:

• IVACAFTOR
Q4: Which of the following is seen in CYSTIC FIBROSIS:

NEET PG JANUARY 2019

A. Low sweat chloride level

B. Elevated sweat chloride

C. Low sweat sodium

D. Elevated sweat potassium level


•Sweat ducts are not
obstructed
•Other ducts are obstructed
due to stasis of secretions
Q5: Which of the following exocrine glandular ducts

is not obstructed in cystic fibrosis ?

NEET 2019 Question

1. Pancreas

2. Lung

3. Sweat glands

4. All if the above


Q5: Which of the following exocrine glandular ducts

is not obstructed in cystic fibrosis ?

NEET 2019 Question

1. Pancreas

2. Lung

3. Sweat glands

4. All if the above


NEET PG 2018 QUESTIONS
1. The abdominal mass swelling of pyloric
stenosis can be best felt by?
NEET 2018
a. Above umbilicus
b. When baby is being fed milk
c. From right to left upper quadrant
d. When baby is sleeping
PEDIATRICS
BY
DR.ANAND
M.D PEDIATRICS
Congenital hypertrophic pyloric stenosis
Doubt kills more dreams than failure
ever will do
• Most common surgical disorder of GI tract
in infant !
•Type of vomiting ?
•Forceful, projectile, post prandial
non bilious vomiting
•Age presentation ?
•2-6 weeks
• 20% symptomatic since birth
• & presentation can be delayed till 5 months
•After vomiting child is hungry
with voracious appetite
•Where else do we see
voracious appetitie ?
•Dehydration, wt loss
•Visible peristalsis from
left to right
•Olive shape mass
1. The abdominal mass swelling of pyloric
stenosis can be best felt by?
NEET 2019
a. Above umbilicus
B. When baby is being fed
c. From right to left upper quadrant
d. When baby is sleeping
1. The abdominal mass swelling of pyloric
stenosis can be best felt by?
NEET 2019
a. Above umbilicus
b. When baby is being fed milk
c. From right to left upper quadrant
d. When baby is sleeping
•Males 2-6 weeks of age
•ABG finding?
• Hypokalemic*** metabolic alkalosis
with paradoxical aciduria
•Most common electrolyte
imbalance seen is ?
•Hypokalemia ( potassium
)
• Most specific & sensitive test ?
•Most specific & sensitive: USG
•Most definitive:
•Most definitive: UGI contrast
•Maternal – drug ?**
• Maternal – Erythromycin**
•What are the contrast findings ?
•String sign
•Double tract sign –
•Shoulder sign
•Mushroom sign
USG : Target sign and antral nipple sign
• Pyloric thickness >4 mm & length
>14 mm (>16 mm Ghai) is diagnostic
•Surgical
treatment ?
•Ramstedt
pyloromyotomy
2 things prevents us from happiness
2 things prevents us from happiness

Living in the past


2 things prevents us from happiness

Living in the past

Observing others
• Identify the Anomaly ?
Identify the abnormality ?
Holoprosencephaly
Q2: Which of the fontanelle is
last to close:
NEET PATTERN JAN 2018

A. Posterior fontanelle

B. Anterior fontanelle

C. Mastoid fontanelle

D. Sphenoidal Fontanelle
• Anterior fontanelle: 10-24mnths
• Posterior: 2-3 months

• Sphenoidal: 6 months
• Mastoid: 6-18 months
Q2: Which of the fontanelle is
last to close:
NEET PATTERN JAN 2018

A. Posterior fontanelle

B. Anterior fontanelle

C. Mastoid fontanelle

D. Sphenoidal Fontanelle
Q3: which of the following is abnormal :

NEET PG JAN 2018

A. RR>40 pm in child 2-12 months of age

B. RR > 50 cpm in a child between 2-12 months of


age

C. RR > 30cpm in a child between 1-5 yrs of age

D. RR > 50cpm in a child <2 months of age


WHO CUT OFF OF FAST BREATHING

•0 – 2 months >60
•3 – 1 year >50
•1-5 yr >40
•>5 yr > 30
Q3: which of the following is abnormal :

NEET PG JAN 2019

A. RR>40 pm in child 2-12 months of age

B. RR > 50 cpm in a child between 3-12


months of age

C. RR > 30cpm in a child between 1-5 yrs of


age

D. RR > 50cpm in a child <2 months of age


Q4) WHAT IS THE NORMAL HC OF A 2 MONTH
OLD IF THE BIRTH HC IS 33 CM ? NEET 2018

a) 34 cm

b) 35 cm

c) 36 cm

d) 37 cm
•What is the normal
head circumference?
•33-35cm
0-3 months 2cm/month

3-6 months 1 cm/ month

7-12 months 0.5 cm/ month

1-3 yr 0.2/month
Q4) WHAT IS THE NORMAL HC OF A 2 MONTH
OLD IF THE BIRTH HC IS 33 CM ? NEET 2018

a) 34 cm

b) 35 cm

c) 36 cm

d) 37 cm
5) SAM IS DEFINED AS ?
Q5: Definition of SAM include which of the

following:
NEET PG JAN 2018

A. Weight for height <-2sd

B. Weight for height <-3sd

C. Weight for age <-2sd

D. Weight of age <-3sd


Q5: Definition of SAM include which of the

following:
NEET PG JAN 2018

A. Weight for height <-2sd

B. Weight for height <-3sd

C. Weight for age <-2sd

D. Weight of age <-3sd


6. Establishment of fetoplacental circulation seen
at? NEET 2018
a. 11 to 13 days
b. 20 to 22 days
c. 7 days
d. 25 to 26 days
•Fetal circulation is established
by ___day after fertilization.
•Fetal circulation is established
by 21st day after fertilization.
5. Establishment of fetoplacental circulation seen
at? NEET 2018
a. 11 to 13 days
b. 20 to 22 days
c. 7 days
d. 25 to 26 days
• The heart assumes its normal four-chambered shape by
the end of __ weeks of intrauterine life.
• The heart assumes its normal four-chambered shape by
the end of 6 weeks of intrauterine life.
Structure

Ductus arteriosus

Umbilical artery

Umbilical vein

Ductus venosus
Structure Remnant

Ductus arteriosus Ligamentum arteriosum

Umbilical artery Medial umbilical ligament

Umbilical vein Ligamentum teres


hepatis

Ductus venosus Ligamentum venosum


6. 1 year child weighing 6 kg is suffering from
acute gastroenteritis along with signs of
sunken eyes and skin pitch goes back within 3
seconds. What will be your management?
a. RL infusion 120 ml in the first hour followed
by 360 ml in the next 5 hours
b. RL infusion 180 ml in the first hour followed
by 420 ml in the next 5 hours
c. RL infusion 180 ml in the first hour followed
by 480 ml in the next 2 hours
d. RL infusion 240 ml in the first hour followed
by 360 ml in the next 5 hours
Assessment of dehydration in patients with diarrhea
Look at MILD MODERATE SEVERE

Condition Well alert Restless, irritable Lethargic or


unconscious; floppy

Eyes Normal Sunken Very sunken and dry

Tears Present Absent Absent

Mouth and Moist Dry Very dry

tongue
Thirst Drinks normally, not Thirsty, drinks ‘Drinks poorly’ or is
thirsty eagerly not able to drink

Feel
Skin pinch Goes back quickly Goes back slowly Goes back very
slowly
Decide The patient has no If the patient has two If the patient has two
signs of dehydration or more signs, there or more sings, there
is some dehydration is severe dehydration

Treat Use treatment Weight the patient, if Weight the patient


Plan A possible, and use and use treatment
treatment Plan C urgently
Plan B
6. 1 year child weighing 6 kg is suffering from
acute gastroenteritis along with signs of
sunken eyes and skin pitch goes back within 3
seconds. What will be your management?
a. RL infusion 120 ml in the first hour followed
by 360 ml in the next 5 hours
b. RL infusion 180 ml in the first hour followed
by 420 ml in the next 5 hours
c. RL infusion 180 ml in the first hour followed
by 480 ml in the next 2 hours
d. RL infusion 240 ml in the first hour followed
by 360 ml in the next 5 hours
•NEET PG
2016- 2017
QUESTIONS
1. What is the age of the child who draws
a circle and builds tower of 9 cubes?
a. 1 year
b. 2 years
c. 2 ½ years
d. 3 years
1. What is the age of the child who draws
a circle and builds tower of 9 cubes?
a. 1 year
b. 2 years
c. 2 ½ years
d. 3 years
2. A 9 month old child with respiratory rate
53/min and presence of cough is classified as:-
a. SIRS
b. Respiratory distress
c. Tachypnea
d. ARDS
•WHO Cut Off of fast
breathing
•WHO Cut Off of fast breathing

•>60
•>50
•>40
•>30
2. A 9 month old child with respiratory rate
53/min and presence of cough is classified as:-
a. SIRS
b. Respiratory distress
c. Tachypnea
d. ARDS
3. Which of the following is not a sign
of severe dehydration?
a. Tachycardia
b. Anuria
c. Increased thirst
d. Delayed capillary refill [>3 sec]
Assessment of dehydration in patients with diarrhea
Look at MILD MODERATE SEVERE

Condition Well alert Restless, irritable Lethargic or


unconscious; floppy

Eyes Normal Sunken Very sunken and dry

Tears Present Absent Absent

Mouth and Moist Dry Very dry

tongue
Thirst Drinks normally, not Thirsty, drinks ‘Drinks poorly’ or is
thirsty eagerly not able to drink

Feel
Skin pinch Goes back quickly Goes back slowly Goes back very
slowly
Decide The patient has no If the patient has two If the patient has two
signs of dehydration or more signs, there or more sings, there
is some dehydration is severe dehydration

Treat Use treatment Weight the patient, if Weight the patient


Plan A possible, and use and use treatment
treatment Plan C urgently
Plan B
•Rx of choice for severe
dehydration is _____
•Rx of choice for severe
dehydration is RL *
3. Which of the following is not a sign
of severe dehydration?
a. Tachycardia
b. Anuria
c. Increased thirst
d. Delayed capillary refill [>3 sec]
4. Anterior fontanelle closes by what age?
a. At term
b. 1 month
c. 18 months
d. 28 months
4. Anterior fontanelle closes by what age?
a. At term
b. 1 month
c. 18 months
d. 28 months
5. Trigonocephaly is due to premature
closure of which suture?
a. Sagittal suture
b. Metopic suture
c. Lambdoid suture
d. Coronal suture
5. Trigonocephaly is due to premature
closure of which suture?
a. Sagittal suture
b. Metopic suture
c. Lambdoid suture
d. Coronal suture
6. Russell silver syndrome is associated with
which of the following?
a. Autosomal inheritance
b. X- linked inheritance
c. Sporadic gene mutation
d. Uniparental disomy
• Male
• Male
• Macrocephaly
• Male
• Macrocephaly
• Limb length asymmetry
Prominent forehead
Prominent forehead
Small triangular
Face
Prominent forehead
Small triangular
Face
Small jaw
Narrow chin
• Bluish sclera
• Bluish sclera
• Defect in chr. 7 & 11 in 50% cases
Clindodactly
• Uniparental disomy refers

to the situation in which 2

copies of a chromosome

come from the same

parent
• Uniparental disomy refers to

the situation in which 2

copies of a chromosome

come from the same parent,

instead of 1 copy coming

from the mother, and 1 copy

coming from the father.


6. Russell silver syndrome is associated with
which of the following?
a. Autosomal inheritance
b. X- linked inheritance
c. Sporadic gene mutation
d. Uniparental disomy
Angelman Syndrome
• Micro puppetshow delay by 15 min. makes
mother hyper

• Thanks to Dr.Nayan
•Maternal gene is deleted

•Paternal gene is doubled


•Maternal gene is deleted

•Paternal gene is doubled

•Dev delay

•Intellectual disability
• Microcephaly
• Hyperactive
• Spontaneous laughter - Happy Puppets
• Hypertonia
• UBE3A GENE ON Chr 15 is
defective/absent * **
Prader Willi Syndrome
• Othername is refrigerator lock syndrome
•Paternal gene is missing /
affected

•Maternal gene is over expressed

•Obesity
•Paternal gene is missing /
affected
•Maternal gene is over expressed
•Obesity
•Small genitalia
•Chronically eating
•Inc. apetitie

•Wt gain

•Microcephaly

•Mental retardation

•Dev delay
Consensus Diagnostic Criteria for Prader-Willi Syndrome

MAJOR CRITERIA (1 point each) MINOR CRITERIA (1/2 point each)


1 Neonatal/infantile hypotonia Decreased fetal movement and infantile lethargy

2 Feeding problems and failure to thrive as an infant Typical behavior problems

3 Weight gain at 1-6 year; obesity; hyperphagia Sleep apnea

4 Characteristic dysmorphic facial features Short stature for family by 15 yr

5 Small genitalia; pubertal delay and insufficiency Hypopigmentation for the family

6 Developmental delay/intellectual disability Small hand and feet for height

7 Narrow hands, straight ulnar border


8 Esotropia, myopia
9 Thick, viscous saliva
10 Speech articulation defects

11 Skin picking
Nutritional Phases in Prader-Willi Syndrome

PHASE MEDIAN AGES CLINICAL CHARACTERISTICS

0 Prenatal to birth Decreased fetal movements and lower birthweight than


siblings

1a 0-9 mo Hypotonia with difficulty feeding and decreased appetite

1b 9-25 mo Improved feeding and appetite and growing appropriately

2a 2.1-4.5 year Weight increasing without appetite increase or excess calories

2b 4.5-8 year Increased appetite and calories, but can feel full

3 8 year to adulthood Hyperphagic, rarely feels full

4 Adulthood Appetite is no longer insatiable


Next syndrome
• Post axial polydactyly
• Obesity
• Pigmentary retinopathy
• Hypogonadism in males
• Mental retardation
• Post axial polydactyly
• Obesity
• Pigmentary retinopathy
• Hypogonadism in males
• Mental retardation

• Bardet biedl sybdrome


7. A newborn after prolonged labour is not breathing well

and after 30 seconds of receiving 100% oxygen by bag and

mask, heart rate is 88 beats per min, what is the next step

in management?

a. Discontinue oxygen and ventilation

b. Discontinue oxygen, continue ventilation

c. Continue oxygen, continue ventilation

d. Start chest compressions


7. A newborn after prolonged labour is not breathing well

and after 30 seconds of receiving 100% oxygen by bag and

mask, heart rate is 88 beats per min, what is the next step

in management?

a. Discontinue oxygen and ventilation

b. Discontinue oxygen, continue ventilation

c. Continue oxygen, continue ventilation

d. Start chest compressions


8. Breast milk protects from infections as it
contains all of the following except:-
a. IgE
b. Lactoferrin
c. Bifidus factor
d. PABA
8. Breast milk protects from infections as it
contains all of the following except:-
a. IgE
b. Lactoferrin
c. Bifidus factor
d. PABA
9. Which of the following is not a
feature of Down’s syndrome?
a. Hypotonia
b. Infections
c. Female infertility
d. Early onset Alzheimer’s disease
9. Which of the following is not a
feature of Down’s syndrome?
a. Hypotonia
b. Infections
c. Female infertility
d. Early onset Alzheimer’s disease
10. Congenital syphilis can be best
diagnosed by-
a. IgM FTA – ABS
b. IgM FTA – ABS
c. VDRL
d. TPI
•O – others syphilis

•(treponema

pallidum, bacteria)

•(3rd trimester)
Syphilis
• Early manifestations:

• Late:
Syphilis
• Early manifestations:

Snuffles*(syphilitic rhinitis)

• Late:
• Snuffles*(syphilitic rhinitis

appears similar to the

rhinitis of the common

cold
• Snuffles*(syphilitic rhinitis

appears similar to the rhinitis

of the common cold

• except it is more severe


• Snuffles*(syphilitic rhinitis

appears similar to the rhinitis

of the common cold

• except it is more severe

• lasts longer
• Snuffles*(syphilitic rhinitis appears

similar to the rhinitis of the

common cold

• except it is more severe

• lasts longer

• bloody rhinorrhea, + - laryngitis.


Syphilis
• Early manifestations:

Snuffles*(syphilitic rhinitis)

• Late: Hutchinson triad*


Syphilis
• Hutchinson teeth:

• Interstitial keratitis

• 8th nerve deafness


Syphilis
• Hutchinson teeth: peg shaped

notched upper central incisor


Mulberry molars: multiple rounded

Rudimentary enamel cusps

on the permanent first


CLUTTONS JOINT PAINLESS JOINT SWELLING-KNEE
•Recurrent abortions
are seen only in ?
•Recurrent abortions are
seen only in ?
•Congenital syphilis
Syphilis
• Diagnosis:
Syphilis
• Diagnosis: IgM-FTA-ABS* NEET

• Immunoglobulin flouroscent

treponemal antibody

absorption
Syphilis
• RX:
Syphilis
• RX: Parenteral penicillin G

• Prenatal screening for syphilis

• Untreated – 100 % mortality


10. Congenital syphilis can be best
diagnosed by-
a. IgM FTA – ABS
b. IgM FTA – ABS
c. VDRL
d. TPI
11. By what are is the milestone of climbing
steps with alternate feet achieved?
a. 2 years
b. 3 years
c. 4 years
d. 5 years
11. By what are is the milestone of climbing
steps with alternate feet achieved?
a. 2 years
b. 3 years
c. 4 years
d. 5 years
12. At what age the first permanent teeth
appear in a child?
a. 2 years
b. 4 years
c. 6 years
d. 8 years
12. At what age the first permanent teeth
appear in a child?
a. 2 years
b. 4 years
c. 6 years
d. 8 years
17. Treatment of choice for acute Kawasaki disease
is-

a. Low molecular weight heparin


b. Aspirin
c. Intravenous immunoglobulin
d. Mycophenolatemofetil
Acute stage
1-2 weeks

Convalescent stage
6-8 week of illness

Long term therapy with


coronary abnormalities

Acute coronary thrombosis


Acute stage IVIG 2g/kg over 10-12 hrs &
1-2 weeks ASPIRIN 80-100 mg/kg/day

QID until pt is afebrile for at

least 48 hrs
Convalescent stage Aspirin 3-5 mg per kg per day
6-8 week of illness once daily until 6-8 week if
normal coronary findings
Long term therapy with Aspirin 3-5 mg per kg once
coronary abnormalities daily or clopidogrel 1mg per
kg per day (max:75mg/day)
Warfarin added for pts at high
risk of thrombosis

Acute coronary thrombosis Fibrinolytic therapy with


tissue plasminogen activator
• Patient receiving IVIG should not be given live
vaccine until 11 months.
17. Treatment of choice for acute Kawasaki disease
is-

a. Low molecular weight heparin


b. Aspirin
c. Intravenous immunoglobulin
d. Mycophenolatemofetil
Q19: Which of the following is not a feature of
minimal change disease?
a. Hypertension
b. Edema
c. Proteinuria
d. Responsive to steroid therapy
Q19: Which of the following is not a feature of
minimal change disease?
a. Hypertension
b. Edema
c. Proteinuria
d. Responsive to steroid therapy
Q20: Most common situation associated with
transient tachypnea of newborn is :-
a. Term delivery requiring forceps
b. Term requiring ventouse
c. Elective caesarean section
d. Normal vaginal delivery
Q20: Most common situation associated with
transient tachypnea of newborn is :-
a. Term delivery requiring forceps
b. Term requiring ventouse
c. Elective caesarean section
d. Normal vaginal delivery
•MC cyanotic CHD in
neonate is
•In infant __________
•Acyanotic always ________
•MC cyanotic CHD in
neonate is TGA
•In infant TOF
•Acyanotic always VSD
❖Your future depends
on what you do today
❖Your future depends on
what you do today
& your today has already
started..
•Thank You

You might also like